Math Problems, Tests, Forums

  • Math Forum/Help
  • Problem Solver
  • College Math
  • Pythagorean Theorem

Pythagorean Theorem: Problems with Solutions

pythagoras theorem problem solving

  • Home   |  
  • About   |  
  • Contact Us   |  
  • Privacy   |  
  • Newsletter   |  
  • Shop   |  
  • 🔍 Search Site
  • Easter Color By Number Sheets
  • Printable Easter Dot to Dot
  • Easter Worksheets for kids
  • Kindergarten
  • All Generated Sheets
  • Place Value Generated Sheets
  • Addition Generated Sheets
  • Subtraction Generated Sheets
  • Multiplication Generated Sheets
  • Division Generated Sheets
  • Money Generated Sheets
  • Negative Numbers Generated Sheets
  • Fraction Generated Sheets
  • Place Value Zones
  • Number Bonds
  • Addition & Subtraction
  • Times Tables
  • Fraction & Percent Zones
  • All Calculators
  • Fraction Calculators
  • Percent calculators
  • Area & Volume Calculators
  • Age Calculator
  • Height Calculator
  • Roman Numeral Calculator
  • Coloring Pages
  • Fun Math Sheets
  • Math Puzzles
  • Mental Math Sheets
  • Online Times Tables
  • Online Addition & Subtraction
  • Math Grab Packs
  • All Math Quizzes
  • 1st Grade Quizzes
  • 2nd Grade Quizzes
  • 3rd Grade Quizzes
  • 4th Grade Quizzes
  • 5th Grade Quizzes
  • 6th Grade Math Quizzes
  • Place Value
  • Rounding Numbers
  • Comparing Numbers
  • Number Lines
  • Prime Numbers
  • Negative Numbers
  • Roman Numerals
  • Subtraction
  • Add & Subtract
  • Multiplication
  • Fraction Worksheets
  • Learning Fractions
  • Fraction Printables
  • Percent Worksheets & Help
  • All Geometry
  • 2d Shapes Worksheets
  • 3d Shapes Worksheets
  • Shape Properties
  • Geometry Cheat Sheets
  • Printable Shapes
  • Coordinates
  • Measurement
  • Math Conversion
  • Statistics Worksheets
  • Bar Graph Worksheets
  • Venn Diagrams
  • All Word Problems
  • Finding all possibilities
  • Logic Problems
  • Ratio Word Problems
  • All UK Maths Sheets
  • Year 1 Maths Worksheets
  • Year 2 Maths Worksheets
  • Year 3 Maths Worksheets
  • Year 4 Maths Worksheets
  • Year 5 Maths Worksheets
  • Year 6 Maths Worksheets
  • All AU Maths Sheets
  • Kindergarten Maths Australia
  • Year 1 Maths Australia
  • Year 2 Maths Australia
  • Year 3 Maths Australia
  • Year 4 Maths Australia
  • Year 5 Maths Australia
  • Meet the Sallies
  • Certificates

Pythagoras Theorem Questions

Welcome to our Pythagoras' Theorem Questions area. Here you will find help, support and questions to help you master Pythagoras' Theorem and apply it.

For full functionality of this site it is necessary to enable JavaScript.

Here are the instructions how to enable JavaScript in your web browser .

Pythagoras' Theorem Questions

Here you will find our support page to help you learn to use and apply Pythagoras' theorem.

Please note: Pythagoras' Theorem is also called the Pythagorean Theorem

There are a range of sheets involving finding missing sides of right triangles, testing right triangles and solving word problems using Pythagoras' theorem.

Using these sheets will help your child to:

  • learn Pythagoras' right triangle theorem;
  • use and apply the theorem in a range of contexts to solve problems.

Pythagoras' Theorem

where a,b and c are the sides of a right triangle.
Side c is the hypotenuse (longest side).

Pythagoras' Theorem - in more detail

Pythagoras' theorem states that in a right triangle (or right-angled triangle) the sum of the squares of the two smaller sides of the triangle is equal to the square of the hypotenuse.

Right triangle labelled

In other words, \[ a^2 + b^2 = c^2 \]

where c is the hypotenuse (the longest side) and a and b are the other sides of the right triangle.

What does this mean?

This means that for any right triangle, the orange square (which is the square made using the longest side) has the same area as the other two blue squares added together.

Right triangle pythagoras

Other formulas that can be deduced from the Pythagorean theorem

As a result of the formula \[ a^2 + b^2 = c^2 \] we can also deduce that:

  • \[ b^2 = c^2 - a^2 \]
  • \[ a^2 = c^2 - b^2 \]
  • \[ c = \sqrt{a^2 + b^2} \]
  • \[ b = \sqrt {c^2 - a^2} \]
  • \[ a = \sqrt {c^2 - b^2} \]

Pythagarean Theorem Examples

Example 1) find the length of the missing side..

Pythagoras theorem example 1

In this example, we need to find the hypotenuse (longest side of a right triangle).

So using pythagoras, the sum of the two smaller squares is equal to the square of the hypotenuse.

This gives us \[ 4^2 + 6^2 = ?^2 \]

So \[ ?^2 = 16 + 36 = 52 \]

This gives us \[ ? = \sqrt {52} = 7.21 \; cm \; to \; 2 \; decimal \; places \]

Example 2) Find the length of the missing side.

Pythagoras example 2

In this example, we need to find the length of the base of the triangle, given the other two sides.

This gives us \[ ?^2 + 5^2 = 8^2 \]

So \[ ?^2 = 8^2 - 5^2 = 64 - 25 = 39 \]

This gives us \[ ? = \sqrt {39} = 6.25 \; cm \; to \; 2 \; decimal \; places \]

Pythagoras' Theorem Question Worksheets

The following questions involve using Pythagoras' theorem to find the missing side of a right triangle.

The first sheet involves finding the hypotenuse only.

A range of different measurement units have been used in the triangles, which are not drawn to scale.

  • Pythagoras Questions Sheet 1
  • PDF version
  • Pythagoras Questions Sheet 2
  • Pythagoras Questions Sheet 3
  • Pythagoras Questions Sheet 4

Pythagoras' Theorem Questions - Testing Right Triangles

The following questions involve using Pythagoras' theorem to find out whether or not a triangle is a right triangle, (whether the triangle has a right angle).

If Pythagoras' theorem is true for the triangle, and c 2 = a 2 + b 2 then the triangle is a right triangle.

If Pythagoras' theorem is false for the triangle, and c 2 = a 2 + b 2 then the triangle is not a right triangle.

  • Pythagoras Triangle Test Sheet 1
  • Pythagoras Triangle Test Sheet 2

Pythagoras' Theorem Questions - Word Problems

The following questions involve using Pythagoras' theorem to solve a range of word problems involving 'real-life' type questions.

On the first sheet, only the hypotenuse needs to be found, given the measurements of the other sides.

Illustrations have been provided to support students solving these word problems.

  • Pythagoras Theorem Word Problems 1
  • Pythagoras Theorem Word Problems 2

Geometry Formulas

  • Geometry Formula Sheet

Here you will find a support page packed with a range of geometric formula.

Included in this page are formula for:

  • areas and volumes of 2d and 3d shapes
  • interior angles of polygons
  • angles of 2d shapes
  • triangle formulas and theorems

This page will provide a useful reference for anyone needing a geometric formula.

Triangle Formulas

Here you will find a support page to help you understand some of the special features that triangles have, particularly right triangles.

Using this support page will help you to:

  • understand the different types and properties of triangles;
  • understand how to find the area of a triangle;
  • know and use Pythagoras' Theorem.

All the free printable geometry worksheets in this section support the Elementary Math Benchmarks.

  • Geometry Formulas Triangles

Here you will find a range of geometry cheat sheets to help you answer a range of geometry questions.

The sheets contain information about angles, types and properties of 2d and 3d shapes, and also common formulas associated with 2d and 3d shapes.

Included in this page are:

  • images of common 2d and 3d shapes;
  • properties of 2d and 3d shapes;
  • formulas involving 2d shapes, such as area and perimeter, pythagoras' theorem, trigonometry laws, etc;
  • formulas involving 3d shapes about volume and surface area.

Using the sheets in this section will help you understand and answer a range of geometry questions.

How to Print or Save these sheets 🖶

Need help with printing or saving? Follow these 3 steps to get your worksheets printed perfectly!

  • How to Print support

Subscribe to Math Salamanders News

Sign up for our newsletter to get free math support delivered to your inbox each month. Plus, get a seasonal math grab pack included for free!

math salamanders news image

  • Newsletter Signup

Return to Geometry Section

Return from Pythagoras Theorem Questions to Math Salamanders Homepage

Math-Salamanders.com

The Math Salamanders hope you enjoy using these free printable Math worksheets and all our other Math games and resources.

We welcome any comments about our site or worksheets on the Facebook comments box at the bottom of every page.

New! Comments

TOP OF PAGE

Follow Me on Pinterest

© 2010-2024 Math Salamanders Limited. All Rights Reserved.

  • Privacy Policy
  • Copyright Policy

Word problems on Pythagorean Theorem

Learn how to solve different types of word problems on Pythagorean Theorem .

Pythagoras Theorem can be used to solve the problems step-by-step when we know the length of two sides of a right angled triangle and we need to get the length of the third side.

Three cases of word problems on Pythagorean Theorem :

Case 1: To find the hypotenuse where perpendicular and base are given.

Case 2: To find the base where perpendicular and hypotenuse are given.

Case 3: To find the perpendicular where base and hypotenuse are given.

Word problems using the Pythagorean Theorem:

1. A person has to walk 100 m to go from position X in the north of east direction to the position B and then to the west of Y to reach finally at position Z. The position Z is situated at the north of X and at a distance of 60 m from X. Find the distance between X and Y.

Let XY = x m

Therefore, YZ = (100 – x) m

In ∆ XYZ, ∠Z = 90°


Therefore, by Pythagoras theorem

XY = YZ + XZ

⇒ x = (100 – x) + 60

⇒ = 10000 – 200x + + 3600

Pythagorean Theorem Word Problem

⇒ 200x = 10000 + 3600

⇒ 200x = 13600

⇒ x = 13600/200

Therefore, distance between X and Y = 68 meters.

Word problems on Pythagorean Theorem

Therefore, length of each side is 8 cm.

Using the formula solve more word problems on Pythagorean Theorem.

3. Find the perimeter of a rectangle whose length is 150 m and the diagonal is 170 m.

Word problem on Pythagorean Theorem

In a rectangle, each angle measures 90°.

Therefore PSR is right angled at S

Using Pythagoras theorem, we get

⇒ PS = √6400

Therefore perimeter of the rectangle PQRS = 2 (length + width)

                                                          = 2 (150 + 80) m

                                                          = 2 (230) m

                                                          = 460 m

4. A ladder 13 m long is placed on the ground in such a way that it touches the top of a vertical wall 12 m high. Find the distance of the foot of the ladder from the bottom of the wall.

Word problems using the Pythagorean Theorem

Let the required distance be x meters. Here, the ladder, the wall and the ground from a right-angled triangle. The ladder is the hypotenuse of that triangle.

According to Pythagorean Theorem,

Therefore, distance of the foot of the ladder from the bottom of the wall = 5 meters.

5. The height of two building is 34 m and 29 m respectively. If the distance between the two building is 12 m, find the distance between their tops.

Pythagorean Theorem: Word Problems

The vertical buildings AB and CD are 34 m and 29 m respectively.

Draw DE ┴ AB

Then AE = AB – EB but EB = BC

Therefore AE = 34 m - 29 m = 5 m

Now, AED is right angled triangle and right angled at E.

⇒ AD = √169

Therefore the distance between their tops = 13 m.

The examples will help us to solve various types of word problems on Pythagorean Theorem.

Congruent Shapes

Congruent Line-segments

Congruent Angles

Congruent Triangles

Conditions for the Congruence of Triangles

Side Side Side Congruence

Side Angle Side Congruence

Angle Side Angle Congruence

Angle Angle Side Congruence

Right Angle Hypotenuse Side congruence

Pythagorean Theorem

Proof of Pythagorean Theorem

Converse of Pythagorean Theorem

7th Grade Math Problems 8th Grade Math Practice From Word problems on Pythagorean Theorem to HOME PAGE

Didn't find what you were looking for? Or want to know more information about Math Only Math . Use this Google Search to find what you need.

New! Comments

Share this page: What’s this?

  • Preschool Activities
  • Kindergarten Math
  • 1st Grade Math
  • 2nd Grade Math
  • 3rd Grade Math
  • 4th Grade Math
  • 5th Grade Math
  • 6th Grade Math
  • 7th Grade Math
  • 8th Grade Math
  • 9th Grade Math
  • 10th Grade Math
  • 11 & 12 Grade Math
  • Concepts of Sets
  • Probability
  • Boolean Algebra
  • Math Coloring Pages
  • Multiplication Table
  • Cool Maths Games
  • Math Flash Cards
  • Online Math Quiz
  • Math Puzzles
  • Binary System
  • Math Dictionary
  • Conversion Chart
  • Homework Sheets
  • Math Problem Ans
  • Free Math Answers
  • Printable Math Sheet
  • Funny Math Answers
  • Employment Test
  • Math Patterns
  • Link Partners
  • Privacy Policy
E-mail Address
First Name

to send you Math Only Math.

Recent Articles

Bisecting an angle | bisecting an angle by using a protractor | rules.

Aug 24, 24 04:14 PM

Construction of Angles by using Compass, Construction of Angles

Aug 24, 24 03:30 PM

Construction of an Angle using a Protractor | Drawing an Angle

Aug 24, 24 02:26 PM

Construction of Perpendicular Bisector of a Line Segment|Steps-by-Step

Aug 24, 24 11:35 AM

Perpendicular Bisector with the Help of Transparent Tapes

5th Grade Geometry Worksheet | Angles | Triangles | Classification

Aug 23, 24 02:58 PM

Find the Number of Triangles

© and ™ math-only-math.com. All Rights Reserved. 2010 - 2024.

Pythagorean Theorem

$a$

This is generalized by the Pythagorean Inequality and the Law of Cosines .

  • 1.1 Proof 1
  • 1.2 Proof 2
  • 1.3 Proof 3
  • 2 Common Pythagorean Triples
  • 3.1 Introductory
  • 3.2.1 Solution 1 (Bash)
  • 3.2.2 Solution 2 (Using 3-4-5)
  • 3.3.1 Solution (Casework)
  • 4 External links

$ABC$

Common Pythagorean Triples

$a^{2}+b^{2}=c^{2}$

Also, if (a,b,c) are a pythagorean triplet it follows that (ka,kb,kc) will also form a pythagorean triplet for any constant k.

k can also be imaginary.

Introductory

  • 2006 AIME I Problem 1
  • 2007 AMC 12A Problem 10

Sample Problem

$333$

Solution 1 (Bash)

$\sqrt{333^2 + 444^2} = 555$

Solution 2 (Using 3-4-5)

$333-444$

Another Problem

$3$

Solution (Casework)

3 and 4 are the legs. Then 5 is the hypotenuse.

3 is a leg and 4 is the hypotenuse.

There are no more cases as the hypotenuse has to be greater than the leg.

$4+5=9$

External links

  • 122 proofs of the Pythagorean Theorem

Something appears to not have loaded correctly.

Click to refresh .

pythagoras theorem problem solving

  • Maths Questions

Pythagoras Theorem Questions

Class Registration Banner

Pythagoras theorem questions with detailed solutions are given for students to practice and understand the concept. Practising these questions will be a plus point in preparation for examinations. Let us discuss in brief about the Pythagoras theorem.

Pythagoras’ theorem is all about the relation between sides of a right-angled triangle. According to the theorem, the hypotenuse square equals the sum of squares of the perpendicular sides.

= (Perpendicular) + (Base)

Click here to learn the proof of Pythagoras’ Theorem .

Video Lesson on Pythagoras Theorem

pythagoras theorem problem solving

Pythagoras Theorem Questions with Solutions

Now that we have learnt about the Pythagoras Theorem, lets apply the same by solving the following questions.

Question 1: In a right-angled triangle, the measures of the perpendicular sides are 6 cm and 11 cm. Find the length of the third side.

Let ΔABC be the triangle, right-angled at B, such that AB and BC are the perpendicular sides. Let AB = 6 cm and BC = 11 cm

Then, by the Pythagoras theorem,

AC 2 = AB 2 + BC 2

\(\begin{array}{l}\Rightarrow AC=\sqrt{(AB^{2}+BC^{2})}=\sqrt{6^{2}+11^{2}}\end{array} \)

\(\begin{array}{l}=\sqrt{36+121}=\sqrt{157}\end{array} \)

∴ AC = √157 cm.

Question 2: A triangle is given whose sides are of length 21 cm, 20 cm and 29 cm. Check whether these are the sides of a right-angled triangle.

If these are the sides of a right-angled triangle, it must satisfy the Pythagoras theorem.

We have to check whether 21 2 + 20 2 = 29 2

Now, 21 2 + 20 2 = 441 + 400 = 841 = 29 2

Thus, the given triangle is a right-angled triangle.

If three integers a, b and c are such that a + b = c , then (a, b, c) is called Pythagorean triples.

For any given integer m, (m – 1, 2m, m + 1) is the Pythagorean triplet.

Learn more about .

Question 3: Find the Pythagorean triplet with whose one number is 6.

Now, m 2 + 1 = 9 + 1 = 10

and m 2 – 1 = 9 – 1 = 8

Therefore, the Pythagorean triplet is (6, 8, 10).

Question 4: The length of the diagonal of a square is 6 cm. Find the sides of the square.

Let ABCD be the square, and let AC be the diagonal of length 6 cm. Then triangle ABC is the right-angled triangle such that AB = BC (∵ all sides of a square are equal)

By Pythagoras theorem,

⇒ AC 2 = 2AB 2

⇒ AC = √2 AB

⇒ AB = (1/√2) AC = (1/√2)6 = 3√2 cm.

Question 5: A ladder is kept at a distance of 15 cm from the wall such that the top of the ladder is at the height of 8 cm from the bottom of the wall. Find the length of the wall.

Let AB be the ladder of length x.

Pythagoras theorem question 5

AC 2 + BC 2 = AB 2

\(\begin{array}{l}\Rightarrow AB=\sqrt{AC^{2}+BC^{2}}\end{array} \)

\(\begin{array}{l}\Rightarrow x=\sqrt{8^{2}+15^{2}}=\sqrt{64+225}\end{array} \)

⇒ x = 17 cm

∴ Length of the ladder is 17 cm.

Question 6: Find the area of a rectangle whose length is 144 cm and the length of the diagonal 145 cm.

Let the rectangle be ABCD

Pythagoras theorem question 6

\(\begin{array}{l}\Rightarrow AD=\sqrt{AC^{2}-CD^{2}}=\sqrt{145^{2}-144^{2)}\end{array} \)

⇒ AD = √(21025 – 20736) = √289

⇒ AD = 17 cm

Thus, area of the rectangle ABCD = 17 × 144 = 2448 cm 2 .

  • Properties of Triangles
  • Congruence of Triangles
  • Similar Triangles
  • Trigonometry

Question 7: A boy travels 24 km towards east from his house, then he turned his left and covers another 10 km. Find out his total displacement?

Let the boy’s house is at point O, then to find the total displacement, we have to find OB.

Pythagoras theorem question 7

Clearly, ΔOAB is a right-angled triangle, by Pythagoras theorem,

\(\begin{array}{l} OB=\sqrt{OA^{2}+AB^{2}}=\sqrt{24^{2}-10^{2}}\end{array} \)

⇒ OB = √(576 + 100) = √676

⇒ OB = 26 km.

Question 8: Find the distance between a tower and a building of height 65 m and 34 m, respectively, such that the distance between their top is 29 m.

The figure below shows the situation. Let x be the distance between the tower and the building.

Pythagoras theorem question 8

In right triangle DCE, by Pythagoras theorem,

CE = √(DE 2 – DC 2 ) = √(29 2 – 21 2 )

⇒ x = √(841 – 441) = √400

⇒ x = 20 m.

∴ the distance between the tower and the building is 20 m.

Question 9: Find the area of the triangle formed by the chord of length 10 cm of the circle whose radius is 13 cm.

Let AB be the chord of the circle with the centre at O such that AB = 10 and OA = OB = 13. Draw a perpendicular OM on AB.

Pythagoras theorem question 9

By the property of circle, perpendicular dropped from the centre of the circle on a chord, bisects the chord.

Then, AM = MB = 5 cm.

Now, in right triangle OMB,

OB 2 = OM 2 + MB 2

⇒ OM = √(OB 2 – MB 2 )

⇒ OM = √(13 2 – 5 2 ) = √(169 – 25)

⇒ OM = √144 = 12 cm

Area of triangle OAB = ½ × AB × OM

= ½ × 10 × 12

= 60 cm 2 .

Question 10: Find the length of tangent PT where P is a point which is at a distance 10 cm from the centre O of the circle of radius 6 cm.

Given, OP = 10 cm and OT = 6m.

We have to find the value of PT.

Pythagoras theorem question 10

By the property of tangents, the radius of the circle is perpendicular to the tangent at the point of contact.

Thus, triangle OTP is a right-angled triangle.

∴ by the Pythagoras theorem,

OP 2 = OT 2. + PT 2

⇒ PT = √(OP 2 – OT 2 ) = √(10 2 – 6 2 )

⇒ PT = √(100 – 36) = √64

⇒ PT = 8 cm.

Related Video on Pythagorean Triples

pythagoras theorem problem solving

Practice Questions on Pythagoras Theorem

1. Find the area of a right-angled triangle whose hypotenuse is 13 cm and one of the perpendicular sides is 5 cm.

2. Find the Pythagorean triplet whose one member is 15.

3. Find the perimeter of a rectangle whose diagonal is 5 cm and one of its sides is 4 cm.

4 if a pole of length 65 cm is kept leaning against a wall such that the pole reaches up to a height of 63 cm on the wall from the ground. Find the distance between the pole and the wall.

5. Find the area of the triangle inscribed within a circle of radius 8.5 cm such that one of the sides of the triangle is the diameter of the circle and the length of the other side is 8 cm.

(Hint: The triangle is formed in semi-circular region and angle of a semi-circle is of 90 o )

Learn about various mathematical concepts in a simple manner with detailed information, along with step by step solutions to all questions, only at BYJU’S. Download BYJU’S – The Learning App to get personalised videos.

MATHS Related Links

pythagoras theorem problem solving

Register with BYJU'S & Download Free PDFs

Register with byju's & watch live videos.

High Impact Tutoring Built By Math Experts

Personalized standards-aligned one-on-one math tutoring for schools and districts

In order to access this I need to be confident with:

Powers and roots

Pythagorean Theorem

Here you will learn about the Pythagorean Theorem, including how to find side measurements of a right triangle and using Pythagoras’ theorem to check and see if a triangle has a right angle or not.

Students will first learn about Pythagorean Theorem as part of geometry in 8 th grade and continue to use it in high school.

What is the Pythagorean Theorem?

The Pythagorean Theorem states that the square of the longest side of a right triangle (called the hypotenuse) is equal to the sum of the squares of the other two sides.

Pythagorean Theorem formula shown with triangle ABC is:

a^2+b^2=c^2

US Webpages_ Pythagorean Theorem 1 US

Side c is known as the hypotenuse . The hypotenuse is the longest side of a right triangle. Side a and side b are known as the adjacent sides. They are adjacent, or next to, the right angle.

You can only use the Pythagorean Theorem with right triangles.

For example,

Let’s look at this right triangle:

US Webpages_ Pythagorean Theorem 2 US

Above, three square grids have been drawn next to each of the sides of the triangle.

The area of the side of length 3=3 \times 3=3^2=9

The area of the side of length 4=4 \times 4=4^2=16

The area of the side of length 5=5 \times 5=5^2=25

The sum of the areas of the squares on the two shorter sides is equal to the area of the square on the longest side.

When you square the sides of the two shorter sides of a right triangle and add them together, you get the square of the longest side.

3^2+4^2=5^2

3, 4, 5 is known as a Pythagorean triple.

There are other Pythagorean triples such as 5, 12, 13 and 8, 15, 17.

If you know two lengths of a right triangle, you can use Pythagorean Theorem to work out the length of the third side.

Pythagorean Theorem Proof

Use the drawing of a square with a smaller square shown inside with the proof below.

US Webpages_ Pythagorean Theorem 3 US

The area of each triangle is \cfrac{1}{2} \, a b and the area of the smaller square is c^2.

There are two ways to find the area of the larger square.

  • Combine the area of the four congruent triangles and the smaller square: \begin{aligned}& =\cfrac{1}{2} \, a b+\cfrac{1}{2} \, a b+\cfrac{1}{2} \, a b+\cfrac{1}{2} \, a b+c^2 \\\\ & =2 a b+c^2\end{aligned}
  • Multiply the side lengths of the larger square together: \begin{aligned}& =(a+b)(a+b) \\\\ & =a^2+2 a b+b^2\end{aligned}

Now set the two expressions equal to each other to prove a^2+b^2=c^2 \text{:}

\begin{aligned}a^2 +2 a b+b^2 &=2 a b+c^2 \\\\ -2 a b \hspace{0.3cm} & \hspace{0.3cm} -2 a b \\\\ a^2+b^2&=c^2\end{aligned}

Since the triangles formed by the vertices of a square will also be right triangles, the proof above shows that a^2+b^2=c^2 will always be true for the sides of a right triangle.

[FREE] Pythagorean Theorem Worksheet (Grades 8)

[FREE] Pythagorean Theorem Worksheet (Grades 8)

Use this quiz to check your grade 8 students’ understanding of pythagorean theorem. 15+ questions with answers covering a range of 8th grade topics on pythagorean theorem to identify areas of strength and support!

3D Pythagorean Theorem

You can find the length AG in the cuboid ABCDEFGH using the Pythagorean Theorem.

US Webpages_ Pythagorean Theorem 4 US

You can make a right triangle ACG which you can use to calculate AG.

US Webpages_ Pythagorean Theorem 5 US

In order to use Pythagoras’ Theorem, you need to know two sides of the triangle. So in order to figure out the longest side AG, you first need to figure out one of the shorter sides AC.

Let’s call this side x and redraw this triangle.

US Webpages_ Pythagorean Theorem 6 US

You can see that the side labeled x forms the diagonal line of the base of the rectangular prism.

US Webpages_ Pythagorean Theorem 7 US

Triangle ABC is a right triangle, so we can use the Pythagorean Theorem to calculate x.

x=\sqrt{10^2 + 4^2} = 2\sqrt{29} = 10.7703…

AG=\sqrt{10.7703…^2 + 6^2} = \sqrt{152}=2\sqrt{38} = 12.328…

So the required length is 12.4 \, cm (rounded to the nearest tenth).

Step-by-step guide: 3D Pythagorean Theorem

What is the Pythagorean Theorem?

Common Core State Standards

How does this relate to 8 th grade math?

  • Grade 8 – Geometry (8.G.B.6) Explain a proof of the Pythagorean Theorem and its converse.
  • Grade 8 – Geometry (8.G.B.7) Apply the Pythagorean Theorem to determine unknown side lengths in right triangles in real-world and mathematical problems in two and three dimensions.

How to use Pythagorean Theorem

In order to use Pythagorean Theorem:

Label the sides of the triangle.

Write down the formula and apply the numbers.

Record the answer.

Pythagorean Theorem examples

Example 1: missing length of the hypotenuse c.

Find x and answer to the nearest hundredth.

US Webpages_ Pythagorean Theorem 8 US

Label the hypotenuse (the longest side) with c. The adjacent sides, next to the right angle can be labeled a and b (either way – they are interchangeable).

US Webpages_ Pythagorean Theorem 9 US

2 Write down the formula and apply the numbers.

\begin{aligned}& a^2+b^2=c^2 \\\\ & 3^2+8^2=x^2 \\\\ & 9+64=x^2 \\\\ & 73=x^2 \\\\ & \sqrt{73}=x\end{aligned}

An alternative method of rearranging the formula and to put one calculation into a calculator will also work.

\begin{aligned}& a^2+b^2=c^2 \\\\ & c^2=a^2+b^2 \\\\ & c=\sqrt{a^2+b^2} \\\\ & x=\sqrt{3^2+8^2}\end{aligned}

3 Record the answer.

Make sure you give your final answer in the correct form by calculating the square root value, including units where appropriate.

x=\sqrt{73}=8.5440037…

The final answer to the nearest hundredth is:

x=8.54 \mathrm{~cm}

Example 2: missing length c

Find x and answer to the nearest tenth.

US Webpages_ Pythagorean Theorem 10 US

\begin{aligned}& a^2 + b^2 = c^2 \\\\ & 7^2 + 9^2 = x^2 \\\\ & x^2 = 7^2 + 9^2 \\\\ & x^2 = 49+81 \\\\ & x^2 = 130 \\\\ & x = \sqrt{130}\end{aligned}

An alternative method is to rearrange the formula and put one calculation into a calculator.

\begin{aligned}& a^2+b^2=c^2 \\\\ & c^2=a^2+b^2 \\\\ & c=\sqrt{a^2+b^2} \\\\ & x=\sqrt{7^2+9^2}\end{aligned}

Make sure you give your final answer in the correct form; including units where appropriate.

x=\sqrt{130}=11.40175.…

The final answer to the nearest tenth is:

x=11.4 \mathrm{~cm}

Example 3: finding an adjacent side (a short side)

Find x and write your answer to the nearest hundredth.

US Webpages_ Pythagorean Theorem 12 US

\begin{aligned} & a^2 + b^2 = c^2 \\\\ & x^2 + 5^2 = 8^2 \\\\ & x^2+25 = 64 \\\\ & x^2 = 64 - 25 \\\\ & x^2 = 39 \\\\ & x =\sqrt{39}\ \end{aligned}

\begin{aligned}& a^2+b^2=c^2 \\\\ & a^2=c^2-b^2 \\\\ & a=\sqrt{c^2-b^2} \\\\ & x=\sqrt{8^2-5^2}\end{aligned}

Make sure you give your final answer in the correct form, including units where appropriate.

x=\sqrt{39}=6.244997.…

x=6.24 \mathrm{~cm}

Example 4: finding an adjacent side (a short side)

Find x and write your answer to the nearest tenth.

US Webpages_ Pythagorean Theorem 14 US

\begin{aligned} & a^2 + b^2 = c^2 \\\\ & x^2 + 11^2 = 20^2\\\\ & x^2+121 = 400 \\\\ & x^2= 400 - 121 \\\\ & x^2 = 279\\\\ & x =\sqrt{279}\end{aligned}

\begin{aligned}& a^2+b^2=c^2 \\\\ & a^2=c^2-b^2 \\\\ & a=\sqrt{c^2-b^2} \\\\ & x=\sqrt{20^2-11^2}\end{aligned}

x=\sqrt{279}=16.70329.…

The final answer rounded to the nearest tenth is:

x=16.7 \mathrm{~cm}

Example 5: checking if a triangle has a right angle

Is the triangle below a right triangle?

US Webpages_ Pythagorean Theorem 16 US

Label the longest side with c. The adjacent sides, next to the right angle can be labeled a and b (either way – they are interchangeable).

US Webpages_ Pythagorean Theorem 17 US

\begin{aligned} & a^2 + b^2 = c^2 \\\\ & 8^2 + 10^2 = 13^2 \\\\ & 64+100 = 169 \\\\ & 164 = 169\end{aligned}

But this is NOT correct. Pythagorean Theorem only works with right triangles.

Because 8^2 + 10^2 ≠ 13^2 , the sides of the triangles do not fit with Pythagorean Theorem. Therefore, the triangle is NOT a right triangle and c is not a hypotenuse.

Example 6: checking if a triangle has a right angle

US Webpages_ Pythagorean Theorem 18 US

\begin{aligned}& a^2 + b^2 = c^2 \\\\ & 6^2 + 8^2 = 10^2 \\\\ & 36+64 = 100 \\\\ & 100 = 100\end{aligned}

This is correct. Pythagorean Theorem only works with right triangles.

Because 6^2 + 8^2 = 10^2 , the sides of the triangles fit with Pythagorean Theorem. Therefore, the triangle is a right triangle and c is a hypotenuse.

Teaching tips for Pythagorean Theorem

  • In the beginning, give examples that are on grids and have the sides of the right triangle as positive integers. This allows students to draw the corresponding square for each side of the triangle and test to see that a^2+b^2=c^2.
  • Give students a chance to try and prove the Pythagorean Theorem on their own. Then give students examples of a few different theorems and challenge them to find the one that makes the most sense for them.
  • The Pythagorean Theorem has connections outside of math classes. If time allows, students can explore the history of this theorem.

Easy mistakes to make

  • Not correctly identifying the hypotenuse It is very important to make sure that the hypotenuse, the long side, is correctly identified and labeled c.

US Webpages_ Pythagorean Theorem 20 US

  • Thinking the lengths of sides can only be whole numbers Lengths can be decimals, fractions or even irrational numbers such as, \sqrt{2}.

US Webpages_ Pythagorean Theorem 21 US

  • Rounding too early If you need to use Pythagorean Theorem in a question with multiple steps, do not round until the very end of the question or you will lose accuracy. For example, you may need to find the height of a triangle, and then use that height to find its area.

Practice Pythagorean Theorem questions

1. Find side x. Give your answer to the nearest hundredth:

US Webpages_ Pythagorean Theorem 22 US

\begin{aligned}& a^2 + b^2 = c^2 \\\\ & 7^2 + 5^2 = x^2 \\\\ & x^2 = 7^2 + 5^2\\\\ & x^2 = 49+25 \\\\ & x^2 = 74 \\\\& x = \sqrt{74}\\\\ &x = 8.602325…\end{aligned}

x=8.60 \mathrm{~cm}

2. Find side x. Give your answer to the nearest hundredth:

pythagoras theorem problem solving

\begin{aligned}&a^2 + b^2 = c^2 \\\\ & 14^2 + 10^2 = x^2 \\\\ & x^2 = 14^2 + 10^2 \\\\ & x^2 = 196+10 \\\\ & x^2 = 296 \\\\ & x = \sqrt{296} \\\\ & x = 17.20465… \end{aligned}

x=17.20 \mathrm{~cm}

3. Find side x. Give your answer to the nearest hundredth:

US Webpages_ Pythagorean Theorem 26 US

\begin{aligned}& a^2 + b^2 = c^2 \\\\ & x^2 + 10^2 = 18^2\\\\ & x^2+100 = 324 \\\\ & x^2 = 324 – 100 \\\\ & x^2 = 224 \\\\ & x =\sqrt{224} \\\\ & x = 14.96662…\end{aligned}

x=14.97 \mathrm{~cm}

4. Find side x. Give your answer to the nearest hundredth:

US Webpages_ Pythagorean Theorem 28 US

\begin{aligned}& a^2 + b^2 = c^2 \\\\ & x^2 + 2.5^2 = 7.2^2 \\\\ & x^2+6.25 = 51.84 \\\\ & x^2 = 51.84-6.25 \\\\ & x^2 = 45.59 \\\\ & x =\sqrt{45.59} \\\\ & x = 6.75203…\end{aligned}

x=6.75 \mathrm{~cm}

5. Is this a right triangle? Justify your answer with the Pythagorean Theorem.

US Webpages_ Pythagorean Theorem 30 US

No, because 12^2+5^2 ≠ 13^2

Yes, because I measured the angle and it was 90^{\circ}

Yes, because 12^2+5^2=13^2

No, because I measured the angle and it was not 90^{\circ}

US Webpages_ Pythagorean Theorem 31 US

\begin{aligned}& a^2 + b^2 = c^2 \\\\ & 12^2 + 5^2 = 13^2 \\\\ & 144+25 = 169 \\\\ & 169 = 169 \end{aligned}

Therefore the triangle is a right triangle.

6. Is this a right triangle? Justify your answer with the Pythagorean Theorem.

US Webpages_ Pythagorean Theorem 32 US

Yes, because 6^2+13^2=14^2

No, because 6^2+13^2 ≠ 14^2

US Webpages_ Pythagorean Theorem 33 US

\begin{aligned}& a^2 + b^2 = c^2 \\\\ & 6^2 + 13^2 = 14^2 \\\\ & 36+169 = 196 \\\\ & 205 = 196\end{aligned}

This is NOT correct. Pythagorean Theorem only works with right triangles.

Therefore the triangle is NOT a right-angled triangle.

Pythagorean Theorem FAQs

Pythagorean Theorem is named after a Greek mathematician who lived about 2,500 years ago, however, the ancient Babylonians used this rule about 4 thousand years ago! At the same time, the Egyptians were using the theorem to help them with right angles when building structures.

No, this is only true for the sides of a right triangle. The sum of the squares of the lengths for a and b will only be equal to the square of side c if the triangle is right. However, you can use this relationship to decide if a triangle is acute or obtuse. For an acute triangle, the square of the hypotenuse will be less than the sum of the squares of a and b. For an obtuse triangle, the square of the hypotenuse will be more than the sum of the squares of a and b.

Yes, there are many algebraic proofs and geometric proofs that address the Pythagorean Theorem. The proof shown at the top of this page is one of the simplest ways to prove the Pythagorean Theorem.

No, though they all follow the Pythagorean Theorem this does not mean they are similar. They are only similar if there is a multiplicative relationship between each corresponding side of the triangle.

The next lessons are

  • Trigonometry
  • Circle math
  • Sectors, arcs and segments

Still stuck?

At Third Space Learning, we specialize in helping teachers and school leaders to provide personalized math support for more of their students through high-quality, online one-on-one math tutoring delivered by subject experts.

Each week, our tutors support thousands of students who are at risk of not meeting their grade-level expectations, and help accelerate their progress and boost their confidence.

One on one math tuition

Find out how we can help your students achieve success with our math tutoring programs .

[FREE] Common Core Practice Tests (3rd to 8th Grade)

Prepare for math tests in your state with these 3rd Grade to 8th Grade practice assessments for Common Core and state equivalents.

Get your 6 multiple choice practice tests with detailed answers to support test prep, created by US math teachers for US math teachers!

Privacy Overview

Corbettmaths

Pythagoras Practice Questions

Click here for questions, click here for answers.

GCSE Revision Cards

pythagoras theorem problem solving

5-a-day Workbooks

pythagoras theorem problem solving

Primary Study Cards

pythagoras theorem problem solving

Privacy Policy

Terms and Conditions

Corbettmaths © 2012 – 2024

MathBootCamps

The pythagorean theorem with examples.

The Pythagorean theorem is a way of relating the leg lengths of a right triangle to the length of the hypotenuse, which is the side opposite the right angle. Even though it is written in these terms, it can be used to find any of the side as long as you know the lengths of the other two sides. In this lesson, we will look at several different types of examples of applying this theorem.

Table of Contents

  • Examples of using the Pythagorean theorem
  • Solving applied problems (word problems)
  • Solving algebraic problems

[adsenseWide]

Applying the Pythagorean theorem (examples)

In the examples below, we will see how to apply this rule to find any side of a right triangle triangle. As in the formula below, we will let a and b be the lengths of the legs and c be the length of the hypotenuse. Remember though, that you could use any variables to represent these lengths.

image showing the pythagorean theorem formula

In each example, pay close attention to the information given and what we are trying to find. This helps you determine the correct values to use in the different parts of the formula.

Find the value of \(x\).

triangle with unknown hypotenuse

The side opposite the right angle is the side labelled \(x\). This is the hypotenuse. When applying the Pythagorean theorem, this squared is equal to the sum of the other two sides squared. Mathematically, this means:

\(6^2 + 8^2 = x^2\)

Which is the same as:

\(100 = x^2\)

Therefore, we can write:

\(\begin{align}x &= \sqrt{100}\\ &= \bbox[border: 1px solid black; padding: 2px]{10}\end{align}\)

Maybe you remember that in an equation like this, \(x\) could also be –10, since –10 squared is also 100. But, the length of any side of a triangle can never be negative and therefore we only consider the positive square root.

In other situations, you will be trying to find the length of one of the legs of a right triangle. You can still use the Pythagorean theorem in these types of problems, but you will need to be careful about the order you use the values in the formula.

Find the value of \(y\).

example where a leg length is unknown

The side opposite the right angle has a length of 12. Therefore, we will write:

\(8^2 + y^2 = 12^2\)

This is the same as:

\(64 + y^2 = 144\)

Subtracting 64 from both sides:

\(y^2 = 80\)

\(\begin{align}y &= \sqrt{80} \\ &= \sqrt{16 \times 5} \\ &= \bbox[border: 1px solid black; padding: 2px]{4\sqrt{5}}\end{align}\)

In this last example, we left the answer in exact form instead of finding a decimal approximation. This is common unless you are working on an applied problem.

Applications (word problems) with the Pythagorean theorem

There are many different kinds of real-life problems that can be solved using the Pythagorean theorem. The easiest way to see that you should be applying this theorem is by drawing a picture of whatever situation is described.

Two hikers leave a cabin at the same time, one heading due south and the other headed due west. After one hour, the hiker walking south has covered 2.8 miles and the hiker walking west has covered 3.1 miles. At that moment, what is the shortest distance between the two hikers?

First, sketch a picture of the information given. Label any unknown value with a variable name, like x.

figure for a word problem with the pythagorean theorem

Due south and due west form a right angle, and the shortest distance between any two points is a straight line. Therefore, we can apply the Pythagorean theorem and write:

\(3.1^2 + 2.8^2 = x^2\)

Here, you will need to use a calculator to simplify the left-hand side:

\(17.45 = x^2\)

Now use your calculator to take the square root. You will likely need to round your answer.

\(\begin{align}x &= \sqrt{17.45} \\ &\approx 4.18 \text{ miles}\end{align}\)

As you can see, it will be up to you to determine that a right angle is part of the situation given in the word problem. If it isn’t, then you can’t use the Pythagorean theorem.

Algebra style problems with the Pythagorean theorem

There is one last type of problem you might run into where you use the Pythagorean theorem to write some type of algebraic expression. This is something that you will not need to do in every course, but it does come up.

A right triangle has a hypotenuse of length \(2x\), a leg of length \(x\), and a leg of length y. Write an expression that shows the value of \(y\) in terms of \(x\).

Since no figure was given, your first step should be to draw one. The order of the legs isn’t important, but remember that the hypotenuse is opposite the right angle.

algebra problem diagram

Now you can apply the Pythagorean theorem to write:

\(x^2 + y^2 = (2x)^2\)

Squaring the right-hand side:

\(x^2 + y^2 = 4x^2\)

When the problem says “the value of \(y\)”, it means you must solve for \(y\). Therefore, we will write:

\(y^2 = 4x^2 – x^2\)

Combining like terms:

\(y^2 = 3x^2\)

Now, use the square root to write:

\(y = \sqrt{3x^2}\)

Finally, this simplifies to give us the expression we are looking for:

\(y = \bbox[border: 1px solid black; padding: 2px]{x\sqrt{3x}}\)

[adsenseLargeRectangle]

The Pythagorean theorem allows you to find the length of any of the three sides of a right triangle. It is one of those things that you should memorize, as it comes up in all areas of math, and therefore in many different math courses you will probably take. Remember to avoid the common mistake of mixing up where the legs go in the formula vs. the hypotenuse and to always draw a picture when one isn’t given.

Share this:

  • Click to share on Twitter (Opens in new window)
  • Click to share on Facebook (Opens in new window)

Back Home

  • Science Notes Posts
  • Contact Science Notes
  • Todd Helmenstine Biography
  • Anne Helmenstine Biography
  • Free Printable Periodic Tables (PDF and PNG)
  • Periodic Table Wallpapers
  • Interactive Periodic Table
  • Periodic Table Posters
  • Science Experiments for Kids
  • How to Grow Crystals
  • Chemistry Projects
  • Fire and Flames Projects
  • Holiday Science
  • Chemistry Problems With Answers
  • Physics Problems
  • Unit Conversion Example Problems
  • Chemistry Worksheets
  • Biology Worksheets
  • Periodic Table Worksheets
  • Physical Science Worksheets
  • Science Lab Worksheets
  • My Amazon Books

Pythagorean Theorem – Definition, Formula, Problems

Pythagorean Theorem in Math

In mathematic, the Pythagorean theorem states that the square of the hypotenuse of a right triangle is equal to the sum of the squares of its other two sides . Another way of stating the theorem is that the sum of the areas of the squares formed by the sides of a right triangle equals the area of the square whose side is the hypotenuse. The theorem is a key relation in Euclidean geometry. It is named for the Greek philosopher Pythagorus.

Remember: The Pythagorean theorem only applies to right triangles!

Pythagorean Theorem Formula

The formula for the Pythagorean theorem describes the relationship between the sides a and b of a right triangle to its hypotenuse, c . A right triangle is one containing a 90° or right angle. The hypotenuse is the side of the triangle opposite from the right angle (which is the largest angle in a right triangle).

a 2 + b 2 = c 2

Solving for a, b, and c

Rearranging the equation gives the formulas solving for a, b, and c:

  • a = (c 2 – b 2 ) ½
  • b = (c 2 – a 2 ) ½
  • c = (a 2 + b 2 ) ½

How to Solve the Pythagorean Theorem – Example Problems

For example, find the hypotenuse of a right triangle with side that have lengths of 5 and 12.

Start with the formula for the Pythagorean theorem and plug in the numbers for the sides a and b to solve for c .

a 2 + b 2 = c 2 5 2 + 12 2 = c 2 c 2 = 5 2 + 12 2 = 25 + 144 = 169 c2 = 169 c = √169 or 169 ½ = 13

For example, solve for side b of a triangle where a is 9 and the hypotenuse c is 15.

a 2 + b 2 = c 2 9 2 + b 2 = 15 2 b 2 = 15 2 – 9 2 = 225 – 81 = 144 b = √144 = 12

Now, let’s combine a bit of algebra with the geometry. Solve for x where the sides of a right triangle are 5x and 4x +5 and the hypotenuse has a length of 8x -3.

a 2 + b 2 = c 2 (5x) 2 + (4x +5) 2 = (8x-3) 2

The (4x + 5) 2 and (8x -3)2 terms are the squares of binomial expressions. So, expanding the equation gives the following:

25x 2 + (4x +5)(4x +5) = (8x -3)(8x -3) 25x 2 = 16×2 + 20x +20x + 25 = 64x – 24x – 24x + 9

Combine like terms:

41x 2 + 40x + 25 = 64x 2 – 48x + 9

Rewrite the equation and solve for x.

0 = 23x 2 – 88x – 16

Apply the quadratic equation and solve for x:

x = [-b ± √(b 2 -4ac)]/2a x = [-(-88) ± √[-88 2 – 4(23)(-16)] / 2(23) = [88 ± √(7744 + 1472)] / 46 = (88 ± 96) / 46

So, there are two answers:

x = (88 + 96)/46 = 4 and (88 – 96).46 = -4/23

A triangle does not have a negative length for its side, so x is 4.

Plugging in”4″ in place of x, the sides of the right triangle are 20, 21, and 29.

Pythagorean Triples

Pythagorean triples are integers a, b, and c, that represent the sides of a right triangle and satisfy the Pythagorean theorem. Here is the list of Pythagorean triples for integers with values less than 100:

(3, 4, 5), (5, 12, 13), (7, 24, 25), (8, 15, 17), (9, 40, 41), (11, 60, 61), (12, 35, 37), (13, 84, 85), (16, 63, 65), (20, 21, 29), (28, 45, 53), (33, 56, 65), (36, 77, 85), (39, 80, 89), (48, 55, 73), (65, 72, 97)

Proof of the Pythagorean Theorem

There are more proofs for the Pythagorean theorem than for any other theorem in geometry! At least 370 proofs are known. Some of these proofs use the parallel postulate. Some rely on the complementarity of acute angles in a right triangle. Proofs using shearing use the properties of parallelograms.

History – Did Pythagoras Discover the Pythagorean Theorem?

While the Pythagorean theorem takes its name from Pythagorus, he did not discover it. Exactly who gets the credit or whether many different places made the discovery independently is a matter of debate. The Mesopotamians made calculations using the formula as early as 2000 BC, which was over a thousand years before Pythagorus. A papyrus from the Egyptian Middle Kingdom, dating between 2000 and 1786 BC, references a math problem describing Pythagorean triples. The Baudhayana Shulba Sutra from India (dating between the 8th and 5th century BC) lists both Pythagorean triples and the Pythagorean theorem. The “Gougu theorem” from China offers a proof for the Pythagorean theorem, which came into use long before its oldest surviving description from the 1st century BC.

Pythagorus of Samos lived between 570 and 495 BC. While he was not the original person who formulated the Pythagorean theorem, he (or his students) may have introduced its proof to ancient Greece. In any case, his philosophical treatment of math left a lasting impression on the world.

  • Bell, John L. (1999). The Art of the Intelligible: An Elementary Survey of Mathematics in its Conceptual Development . Kluwer. ISBN 0-7923-5972-0.
  • Heath, Sir Thomas (1921). “ The ‘Theorem of Pythagoras ‘”. A History of Greek Mathematics (2 Vols.) (Dover Publications, Inc. (1981) ed.). Oxford: Clarendon Press. ISBN 0-486-24073-8.
  • Maor, Eli (2007). The Pythagorean Theorem: A 4,000-Year History . Princeton, New Jersey: Princeton University Press. ISBN 978-0-691-12526-8.
  • Swetz, Frank; Kao, T. I. (1977). Was Pythagoras Chinese?: An Examination of Right Triangle Theory in Ancient China . Pennsylvania State University Press. ISBN 0-271-01238-2.

Related Posts

Mathwarehouse Logo

Pythagorean Theorem

How to Use The Pythagorean Theorem

The Formula

The picture below shows the formula for the Pythagorean theorem. For the purposes of the formula, side $$ \overline{c}$$ is always the hypotenuse . Remember that this formula only applies to right triangles .

The Pythagorean Theorem

Examples of the Pythagorean Theorem

When you use the Pythagorean theorem, just remember that the hypotenuse is always 'C' in the formula above. Look at the following examples to see pictures of the formula.

Examples of the Pythagorean Theorem

Conceptual Animation of Pythagorean Theorem

Demonstration #1.

More on the Pythagorean theorem

Demonstration #2

Video tutorial on how to use the pythagorean theorem.

Step By Step Examples of Using the Pythagorean Theorem

Example 1 (solving for the hypotenuse).

Use the Pythagorean theorem to determine the length of X.

Example 1

Identify the legs and the hypotenuse of the right triangle .

The legs have length 6 and 8 . $$X $$ is the hypotenuse because it is opposite the right angle.

3, 4, 5 right triangle

Substitute values into the formula (remember 'C' is the hypotenuse).

$ A^2+ B^2= \red C^2 \\ 6^2+ 8^2= \red X^2 $

$A^2+ B^2= \red X^2 \\ 100= \red X^2 \\ \sqrt {100} = \red X \\ 10= \red X $

Example 2 (solving for a Leg)

Example 2

The legs have length 24 and $$X$$ are the legs. The hypotenuse is 26.

10 24 25  right triangle

$ \red A^2+ B^2= C^2 \\ \red x^2 + 24^2= {26}^2 $

$ \red x^2 + 24^2= 26^2 \\ \red x^2 + 576= 676 \\ \red x^2 = 676 - 576 \\ \red x^2 = 100 \\ \red x = \sqrt { 100} \\ \red x = 10 $

Practice Problems

Find the length of X.

Pythagorean Theorem Problem

Remember our steps for how to use this theorem. This problems is like example 1 because we are solving for the hypotenuse .

The legs have length 14 and 48 . The hypotenuse is X.

$ A^2 + B^2 = C^2 \\ 14^2 + 48^2 = x^2 $

Solve for the unknown.

$ 14^2 + 48^2 = x^2 \\ 196 + 2304 = x^2 \\ \sqrt{2500} = x \\ \boxed{ 50 = x} $

right triangle 7, 24, 25

Use the Pythagorean theorem to calculate the value of X. Round your answer to the nearest tenth.

Pythagorean Theorem Problem

Remember our steps for how to use this theorem. This problems is like example 2 because we are solving for one of the legs .

The legs have length 9 and X . The hypotenuse is 10.

$ A^2 + B^2 = C^2 \\ 9^2 + x^2 = 10^2 $

$ 9^2 + x^2 = 10^2 \\ 81 + x^2 = 100 \\ x^2 = 100 - 81 \\ x^2 = 19 \\ x = \sqrt{19} \approx 4.4 $

Use the Pythagorean theorem to calculate the value of X. Round your answer to the nearest hundredth.

Pythagorean Theorem Problem

The legs have length '10' and 'X'. The hypotenuse is 20.

$ A^2 + B^2 = C^2 \\ 10^2 + \red x^2 = 20^2 $

$ 10^2 + \red x^2 = 20^2 \\ 100 + \red x^2 = 400 \\ \red x^2 = 400 -100 \\ \red x^2 = 300 \\ \red x = \sqrt{300} \approx 17.32 $

  • Pythagorean Theorem Shell Problem Cool challenging problem from involving pythagoren theorem, isosceles triangles and pattern recognition
  • Free Triangle Worksheets
  • Pythagorean Theorem Calculator

Ultimate Math Solver (Free) Free Algebra Solver ... type anything in there!

Popular pages @ mathwarehouse.com.

Surface area of a Cylinder

Module 11: Geometry

Using the pythagorean theorem to solve problems, learning outcomes.

  • Use the pythagorean theorem to find the unknown length of a right triangle given the two other lengths

The Pythagorean Theorem is a special property of right triangles that has been used since ancient times. It is named after the Greek philosopher and mathematician Pythagoras who lived around [latex]500[/latex] BCE.

Remember that a right triangle has a [latex]90^\circ [/latex] angle, which we usually mark with a small square in the corner. The side of the triangle opposite the [latex]90^\circ [/latex] angle is called the hypotenuse, and the other two sides are called the legs. See the triangles below.

In a right triangle, the side opposite the [latex]90^\circ [/latex] angle is called the hypotenuse and each of the other sides is called a leg.

Three right triangles are shown. Each has a box representing the right angle. The first one has the right angle in the lower left corner, the next in the upper left corner, and the last one at the top. The two sides touching the right angle are labeled

The Pythagorean Theorem

In any right triangle [latex]\Delta ABC[/latex],

[latex]{a}^{2}+{b}^{2}={c}^{2}[/latex]

where [latex]c[/latex] is the length of the hypotenuse [latex]a[/latex] and [latex]b[/latex] are the lengths of the legs.

A right triangle is shown. The right angle is marked with a box. Across from the box is side c. The sides touching the right angle are marked a and b.

To solve problems that use the Pythagorean Theorem, we will need to find square roots. In Simplify and Use Square Roots we introduced the notation [latex]\sqrt{m}[/latex] and defined it in this way:

[latex]\text{If }m={n}^{2},\text{ then }\sqrt{m}=n\text{ for }n\ge 0[/latex]

For example, we found that [latex]\sqrt{25}[/latex] is [latex]5[/latex] because [latex]{5}^{2}=25[/latex].

We will use this definition of square roots to solve for the length of a side in a right triangle.

Use the Pythagorean Theorem to find the length of the hypotenuse.

Right triangle with legs labeled as 3 and 4.

Step 1. the problem.
Step 2. what you are looking for. the length of the hypotenuse of the triangle
Step 3. Choose a variable to represent it. Let [latex]c=\text{the length of the hypotenuse}[/latex]

Step 4.

Write the appropriate formula.

Substitute.

[latex]{a}^{2}+{b}^{2}={c}^{2}[/latex]

[latex]{3}^{2}+{4}^{2}={c}^{2}[/latex]

Step 5. the equation. [latex]9+16={c}^{2}[/latex]

[latex]25={c}^{2}[/latex]

[latex]\sqrt{25}={c}^{2}[/latex]

[latex]5=c[/latex]

Step 6.

 

[latex]{3}^{2}+{4}^{2}=\color{red}{{5}^{2}}[/latex]

[latex]9+16\stackrel{?}{=}25[/latex]

[latex]25+25\checkmark[/latex]

Step 7. the question. The length of the hypotenuse is [latex]5[/latex].

Use the Pythagorean Theorem to find the length of the longer leg.

Right triangle is shown with one leg labeled as 5 and hypotenuse labeled as 13.

Step 1. the problem.
Step 2. what you are looking for. The length of the leg of the triangle
Step 3. Choose a variable to represent it. Let [latex]b=\text{the leg of the triangle}[/latex]

Label side

Step 4.

Write the appropriate formula. Substitute.

[latex]{a}^{2}+{b}^{2}={c}^{2}[/latex]

[latex]{5}^{2}+{b}^{2}={13}^{2}[/latex]

Step 5. the equation. Isolate the variable term. Use the definition of the square root.

Simplify.

[latex]25+{b}^{2}=169[/latex]

[latex]{b}^{2}=144[/latex]

[latex]{b}^{2}=\sqrt{144}[/latex]

[latex]b=12[/latex]

Step 6.

[latex]{5}^{2}+\color{red}{12}^{2}\stackrel{?}{=}{13}^{2}[/latex]

[latex]25+144\stackrel{?}{=}169[/latex]

[latex]169=169\checkmark[/latex]

Step 7. the question. The length of the leg is [latex]12[/latex].

Kelvin is building a gazebo and wants to brace each corner by placing a [latex]\text{10-inch}[/latex] wooden bracket diagonally as shown. How far below the corner should he fasten the bracket if he wants the distances from the corner to each end of the bracket to be equal? Approximate to the nearest tenth of an inch.

A picture of a gazebo is shown. Beneath the roof is a rectangular shape. There are two braces from the top to each side. The brace on the left is labeled as 10 inches. From where the brace hits the side to the roof is labeled as x.

Step 1. the problem.
Step 2. what you are looking for. the distance from the corner that the bracket should be attached
Step 3. Choose a variable to represent it. Let = the distance from the corner

Step 4.

Write the appropriate formula.

Substitute.

[latex]{a}^{2}+{b}^{2}={c}^{2}[/latex]

[latex]{x}^{2}+{x}^{2}={10}^{2}[/latex]

Step 5. the equation.

Isolate the variable.

Use the definition of the square root.

Simplify. Approximate to the nearest tenth.

[latex]2x^2=100[/latex]

[latex]x^2=50[/latex]

[latex]x=\sqrt{50}[/latex]

[latex]b\approx{7.1}[/latex]

 

Step 6.

[latex]{a}^{2}+{b}^{2}={c}^{2}[/latex]

[latex](\color{red}{7.1})^2+(\color{red}{7.1})^{2}\stackrel{\text{?}}{\approx}{10}^{2}[/latex]

[latex]50.41+50.41=100.82\approx{100}\quad\checkmark[/latex]

Yes.

Step 7. the question. Kelvin should fasten each piece of wood approximately [latex]7.1″[/latex] from the corner.

In the following video we show two more examples of how to use the Pythagorean Theorem to solve application problems.

  • Question ID 146918, 146916, 146914, 146913. Authored by : Lumen Learning. License : CC BY: Attribution
  • Solve Applications Using the Pythagorean Theorem (c only). Authored by : James Sousa (mathispower4u.com). Located at : https://youtu.be/2P0dJxpwFMY . License : CC BY: Attribution
  • Prealgebra. Provided by : OpenStax. License : CC BY: Attribution . License Terms : Download for free at http://cnx.org/contents/[email protected]

Footer Logo Lumen Waymaker

Pythagorean Theorem Word Problems

In these lessons, we will be looking at how to solve different types of word problems using the Pythagorean Theorem.

Related Pages Pythagorean Theorem Converse Of Pythagorean Theorem Applications Of Pythagorean Theorem More Geometry Lessons

How To Solve Word Problems Using The Pythagorean Theorem?

  • Determine whether the word problem can be modeled by a right triangle.
  • Use the Pythagorean Theorem to find the missing side if you are given two sides.

Example: Shane marched 3 m east and 6 m north. How far is he from his starting point?

Solution: First, sketch the scenario. The path taken by Shane forms a right-angled triangle. The distance from the starting point forms the hypotenuse.

Example: The rectangle PQRS represents the floor of a room.

Ivan stands at point A. Calculate the distance of Ivan from a) the corner R of the room b) the corner S of the room

Example: In the following diagram of a circle, O is the centre and the radius is 12 cm. AB and EF are straight lines.

Find the length of EF if the length of OP is 6 cm.

Examples Of Real Life Pythagorean Theorem Word Problems

Problem 1: A 35-foot ladder is leaning against the side of a building and is positioned such that the base of the ladder is 21 feet from the base of the building. How far above the ground is the point where the ladder touches the building?

Problem 2: The main mast of a fishing boat is supported by a sturdy rope that extends from the top of the mast to the deck. If the mast is 20 feet tall and the rope attached to the deck 15 feet away from the base of the mast, how long is the rope?

Problem 3: If an equilateral triangle has a height of 8, find the length of each side.

Problem 4: Two cyclist start from the same location. One cyclist travels due north and the other due east, at the same speed. Find the speed of each in miles per hour if after two hours they are 17sqrt(2) miles apart.

Problem 5: Two cars start from the same intersection with one traveling southbound while the other travels eastbound going 10 mph faster. If after two hours they are 10sqrt(34) apart, how fast was each car traveling?

Problem 6: A carpet measures 7 feet long and has a diagonal measurement of sqrt(74) feet. Find the width of the carpet.

Problem 7: Jim and Eileen decided to take a short cut through the woods to go to their friend’s house. When they went home they decided to take the long way around the woods to avoid getting muddy shoes. What total distance did they walk to and from their friend’s house? Dimensions are in meters.

Problem 8: Shari went to a level field to fly a kite. She let out all 650 feet of the string and tied it to a stake. Then, she walked out on the field until she was directly under the kite, which was 600 feet from the stake. How high was the kite from the ground?

Mathway Calculator Widget

We welcome your feedback, comments and questions about this site or page. Please submit your feedback or enquiries via our Feedback page.

  • + ACCUPLACER Mathematics
  • + ACT Mathematics
  • + AFOQT Mathematics
  • + ALEKS Tests
  • + ASVAB Mathematics
  • + ATI TEAS Math Tests
  • + Common Core Math
  • + DAT Math Tests
  • + FSA Tests
  • + FTCE Math
  • + GED Mathematics
  • + Georgia Milestones Assessment
  • + GRE Quantitative Reasoning
  • + HiSET Math Exam
  • + HSPT Math
  • + ISEE Mathematics
  • + PARCC Tests
  • + Praxis Math
  • + PSAT Math Tests
  • + PSSA Tests
  • + SAT Math Tests
  • + SBAC Tests
  • + SIFT Math
  • + SSAT Math Tests
  • + STAAR Tests
  • + TABE Tests
  • + TASC Math
  • + TSI Mathematics
  • + ACT Math Worksheets
  • + Accuplacer Math Worksheets
  • + AFOQT Math Worksheets
  • + ALEKS Math Worksheets
  • + ASVAB Math Worksheets
  • + ATI TEAS 6 Math Worksheets
  • + FTCE General Math Worksheets
  • + GED Math Worksheets
  • + 3rd Grade Mathematics Worksheets
  • + 4th Grade Mathematics Worksheets
  • + 5th Grade Mathematics Worksheets
  • + 6th Grade Math Worksheets
  • + 7th Grade Mathematics Worksheets
  • + 8th Grade Mathematics Worksheets
  • + 9th Grade Math Worksheets
  • + HiSET Math Worksheets
  • + HSPT Math Worksheets
  • + ISEE Middle-Level Math Worksheets
  • + PERT Math Worksheets
  • + Praxis Math Worksheets
  • + PSAT Math Worksheets
  • + SAT Math Worksheets
  • + SIFT Math Worksheets
  • + SSAT Middle Level Math Worksheets
  • + 7th Grade STAAR Math Worksheets
  • + 8th Grade STAAR Math Worksheets
  • + THEA Math Worksheets
  • + TABE Math Worksheets
  • + TASC Math Worksheets
  • + TSI Math Worksheets
  • + AFOQT Math Course
  • + ALEKS Math Course
  • + ASVAB Math Course
  • + ATI TEAS 6 Math Course
  • + CHSPE Math Course
  • + FTCE General Knowledge Course
  • + GED Math Course
  • + HiSET Math Course
  • + HSPT Math Course
  • + ISEE Upper Level Math Course
  • + SHSAT Math Course
  • + SSAT Upper-Level Math Course
  • + PERT Math Course
  • + Praxis Core Math Course
  • + SIFT Math Course
  • + 8th Grade STAAR Math Course
  • + TABE Math Course
  • + TASC Math Course
  • + TSI Math Course
  • + Number Properties Puzzles
  • + Algebra Puzzles
  • + Geometry Puzzles
  • + Intelligent Math Puzzles
  • + Ratio, Proportion & Percentages Puzzles
  • + Other Math Puzzles

How to Solve Pythagorean Theorem Problems? (+FREE Worksheet!)

In mathematics, the Pythagorean Theorem is the relationship between three sides of a right triangle.

How to Solve Pythagorean Theorem Problems? (+FREE Worksheet!)

Related Topics

  • How Calculate the Area of Trapezoids
  • How to Find the volume and surface area of Rectangular Prisms
  • How to Solve Triangles Problems
  • How to Find Volume and Surface Area of Cubes
  • How to Find the Perimeter of Polygons

Step by step guide to solve Pythagorean Theorem problems

  • We can use the Pythagorean Theorem to find a missing side in a right triangle.

The Absolute Best Books to Ace Pre-Algebra to Algebra II

The Ultimate Algebra Bundle From Pre-Algebra to Algebra II

The pythagorean theorem – example 1:.

Right triangle ABC has two legs of lengths \(9\) cm (AB) and \(12\) cm (AC). What is the length of the third side (BC)?

Use Pythagorean Theorem: \(\color{blue}{a^2+b^2= c^2}\) Then: \(a^2+b^2= c^2 →9^2+12^2= c^2 →81+144=c^2\) \(c^2=225 →\) \(c=\sqrt{225}=15\) \(cm\) → \(c=15 cm\)

The Pythagorean Theorem – Example 2:

Use Pythagorean Theorem: \(\color{blue}{a^2+b^2= c^2}\) Then: \(a^2+b^2= c^2 →8^2+6^2= c^2 →64+36=c^2\) \(c^2=100 →\) \(c=\sqrt{100}=10\) → \(c=10\)

The Pythagorean Theorem – Example 3:

Find the hypotenuse of the following right triangle.

Use Pythagorean Theorem: \(\color{blue}{a^2+b^2= c^2}\) Then: \(a^2+b^2= c^2 →3^2+4^2= c^2 →9+16=c^2\) \(c^2=25 →\) \(c=\sqrt{25}=5\) → \(c=5\)

The Best Book to Help You Ace Pre-Algebra

Pre-Algebra for Beginners The Ultimate Step by Step Guide to Preparing for the Pre-Algebra Test

The pythagorean theorem – example 4:.

Right triangle ABC has two legs of lengths \(6\) cm (AB) and \(8\) cm (AC). What is the length of the third side (BC)?

Use Pythagorean Theorem: \(\color{blue}{a^2+b^2= c^2}\) Then: \(a^2+b^2= c^2 →6^2+8^2= c^2 →36+64=c^2\) \( c^2=100 →\) \(c=\sqrt{100}=10\) \(cm\) → \(c= 10 cm\)

Exercises for Solveing the Pythagorean Theorem

Find the missing side in each right triangle., download pythagorean relationship worksheet.

  • \(\color{blue}{13}\)
  • \(\color{blue}{5}\)
  • \(\color{blue}{15}\)
  • \(\color{blue}{8}\)

The Greatest Books for Students to Ace the Algebra

Pre-Algebra Exercise Book A Comprehensive Workbook + PreAlgebra Practice Tests

Pre-algebra in 10 days the most effective pre-algebra crash course, college algebra practice workbook the most comprehensive review of college algebra, high school algebra i a comprehensive review and step-by-step guide to mastering high school algebra 1, 10 full length clep college algebra practice tests the practice you need to ace the clep college algebra test.

by: Effortless Math Team about 5 years ago (category: Articles , Free Math Worksheets )

Effortless Math Team

Related to this article, more math articles.

  • How to Solve Theoretical Probability?
  • The Ultimate Praxis Core Math Formula Cheat Sheet
  • Best Office Chairs for Online Math Teachers
  • Decoding Discreteness: A Comprehensive Guide to the Probability Mass Function
  • How to Add and Subtract Rational Expressions? (+FREE Worksheet!)
  • What Kind of Math Is on the CBEST Test?
  • How to Solve Word Problems of Writing Variable Expressions
  • Let’s Make Math Education Relevant for Real Life
  • How to Study Math Effectively?
  • Count Lines of Symmetry

What people say about "How to Solve Pythagorean Theorem Problems? (+FREE Worksheet!) - Effortless Math: We Help Students Learn to LOVE Mathematics"?

No one replied yet.

Leave a Reply Cancel reply

You must be logged in to post a comment.

Pre-Algebra Practice Workbook The Most Comprehensive Review of Pre-Algebra

Algebra i practice workbook the most comprehensive review of algebra 1, algebra ii practice workbook the most comprehensive review of algebra 2, algebra i for beginners the ultimate step by step guide to acing algebra i, algebra ii for beginners the ultimate step by step guide to acing algebra ii, pre-algebra tutor everything you need to help achieve an excellent score.

  • ATI TEAS 6 Math
  • ISEE Upper Level Math
  • SSAT Upper-Level Math
  • Praxis Core Math
  • 8th Grade STAAR Math

Limited time only!

Save Over 45 %

It was $89.99 now it is $49.99

Login and use all of our services.

Effortless Math services are waiting for you. login faster!

Register Fast!

Password will be generated automatically and sent to your email.

After registration you can change your password if you want.

  • Math Worksheets
  • Math Courses
  • Math Topics
  • Math Puzzles
  • Math eBooks
  • GED Math Books
  • HiSET Math Books
  • ACT Math Books
  • ISEE Math Books
  • ACCUPLACER Books
  • Premium Membership
  • Youtube Videos

Effortless Math provides unofficial test prep products for a variety of tests and exams. All trademarks are property of their respective trademark owners.

  • Bulk Orders
  • Refund Policy

pythagoras theorem problem solving

  • Parallelogram
  • Quadrilateral
  • Parallelepiped
  • Tetrahedron
  • Dodecahedron
  • Fraction Calculator
  • Mixed Fraction Calculator
  • Greatest Common Factor Calulator
  • Decimal to Fraction Calculator
  • Whole Numbers
  • Rational Numbers
  • Place Value
  • Irrational Numbers
  • Natural Numbers
  • Binary Operation
  • Numerator and Denominator
  • Order of Operations (PEMDAS)
  • Scientific Notation

Table of Contents

Last modified on June 8th, 2024

#ezw_tco-2 .ez-toc-title{ font-size: 120%; ; ; } #ezw_tco-2 .ez-toc-widget-container ul.ez-toc-list li.active{ background-color: #ededed; } chapter outline

Pythagorean theorem.

The Pythagorean Theorem, also known as Pythagoras theorem is a mathematical relation between the 3 sides of a right triangle, a triangle in which one of 3 angles is 90°. It was discovered and named after the Greek philosopher and mathematician of Samos, Pythagoras.

Does Pythagorean Theorem Work on All Triangles

No, the Pythagorean Theorem works only for right triangles. Thus, it helps to test whether a triangle is right triangle or not.  The theorem is also used to find the length of one side of a right triangle when the other two sides are known.

The theorem was first evident on a 4000-year-old Babylonian tablet (beginning about 1900 B.C.) now known as Plimpton 322. Much later in 570–500/490 BCE, the relationship was popularized when Pythagoras stated it explicitly.

When a triangle has a right angle (90°) and squares are made on each of the 3 sides, then the biggest square has an area equal to the sum of the areas of the other 2 squares.

Thus, the Pythagorean Theorem states that the area of the square formed by the longest side of the right triangle (the hypotenuse) is equal to the sum of the area of the squares formed by the other two sides.

pythagoras theorem problem solving

In the above figure,

Area of □ A = a 2

Area of □ B = b 2

Area of □ C = c 2

pythagoras theorem problem solving

Now, according to the Pythagorean Theorem,

Area of □ A + Area of □ B = Area of □ C

(a × a) + (b × b) = (c × c)

a 2 + b 2 = c 2

The above relation is useful to find an unknown side of a right triangle when the lengths of the other 2 sides are known.

The equation that represents the Pythagorean Theorem in mathematical form is given below:

pythagoras theorem problem solving

Let us find out how it works using an example.

Consider a right triangle with side lengths 3, 4, 5.

pythagoras theorem problem solving

In the above figure, let us use the Pythagorean Theorem

3 2 + 4 2 = 5 2

=> 9 + 16 =25

9, 16, & 25 are the areas of the three squares

This proves how the Pythagorean Theorem works

Let us solve an example to understand the concept better

Find the hypotenuse of a right-angle triangle with the other 2 sides 15 cm and 20 cm.

As we know, In a right angle triangle, a 2 + b 2 = c 2 , here c = length of the hypotenuse, a = 15 cm, c = 20 cm => 15 2 + 20 2 = c 2 => 225 + 400 = c 2 => c 2 = 625 => c = 25 cm Thus, the hypotenuse of a right triangle with sides 15 cm and 20 cm is 25 cm

pythagoras theorem problem solving

As we know, In a right angle triangle, a 2 + b 2 = c 2 , here c = 5, a = 3 cm => 3 2 + b 2 = 5 2 => b 2 = 5 2 – 3 2 => b 2 = 25 – 9 => b 2 = 16 => b = 4 cm

pythagoras theorem problem solving

As we know, In a right angle triangle, a 2 + b 2 = c 2 , here a = 9, b = 12, c = 15 cm => 9 2 + 12 2 => 81 + 144 => 225 As, 15 2 = 225 Thus, the above relation holds true for the Pythagorean Theorem, thus it is a right angle triangle.

Does an 8, 15, 16 triangle have a Right Angle?

As we know, In a right angle triangle, a 2 + b 2 = c 2 Here, => 8 2 + 15 2 => 64 + 225 => 289 But, 16 2 = 256 Thus, the above relation does not hold true for the Pythagorean Theorem, thus it is not a right-angle triangle.

The Pythagorean Theorem can be proved in many ways. The 2 most common ways of proving the theorem are described below:

Algebraic Method

This method helps us to prove the Pythagorean Theorem by using the side lengths.

Let us consider 4 right triangles with side lengths a, b, & c, where c is the length of the hypotenuse and ‘a’ and ‘b’ are the lengths of the other 2 sides

pythagoras theorem problem solving

If we arrange the 4 right triangles in a square of length (a + b), we can derive the equation of the Pythagorean Theorem as shown below:

Area of □ EFGH = c × c = c 2

Area of □ ABCD = Area of □ EFGH + Area of 4 right △s

=> (a + b) 2 = c 2 + 4(1/2 × b × a)

=> (a + b) 2 = c 2 + 2ab

=> a 2 + b 2 + 2ab = c 2 + 2ab

=> a 2 + b 2 = c 2

This proves the Pythagorean Theorem

Using Similar Triangles

pythagoras theorem problem solving

As we know,

△ADB ~ △ABC

∴ CD/CB = CB/CA (Corresponding Sides of Similar Triangles are Equal)

=> CB 2 = CD × CA …….. (1)

△BDA ~ △CBA

∴ AD/BA = BA/CA (Corresponding Sides of Similar Triangles are Equal)

=> BA 2 = AD × CA …….. (2)

Adding equations (1) and (2), we get

CB 2 + BA 2 = (CD × CA) + (AD × CA)

=> CB 2 + BA 2 = CA (CD + AD)

Since, CD + AD = CA

∴ CA 2 = CB 2 + BA 2

Applications: What is It Used For

Through learning the basic concepts of the Pythagorean Theorem is important to determine whether a triangle is a right triangle or not. But, we are even more curious in understanding the applications of the Pythagorean Theorem.

Some common real-life applications of the Pythagorean Theorem are given below:

  • Two-dimensional navigation where it is used for calculate the distance between the ship and the point of navigation. The same principles are also used for air navigation. For example, a plane can use its height above the ground and its distance from the source to its destination.
  • Cartographers use to calculate and survey the numerical distances and heights between different points before a map is drawn. It is also used to calculate the steepness of slopes of hills or mountains.
  • Given 2 straight lines, it is used for calculating the length of the diagonal line connecting them. This application is frequently used in architecture , woodworks , or other physical construction projects. It is also used for laying out square angles for making design in buildings.
  • In oceanography, it is used for calculating speed of sound waves in water.
  • Making of tv screens, computer screens , and solar panels
  • Works to determine the height of ladder required to painta wall of certain height
  • Converse of Pythagoras Theorem
  • Pythagorean Triples
  • Pythagorean Identities

One thought on “ Pythagorean Theorem ”

Correction done

Leave a comment Cancel reply

Your email address will not be published. Required fields are marked *

Save my name, email, and website in this browser for the next time I comment.

Related Materials

pythagoras theorem problem solving

  • Privacy Policy
  • Trigonometry

Join Our Newsletter

© 2024 Mathmonks.com . All rights reserved. Reproduction in whole or in part without permission is prohibited.

Pythagorean Theorem

The pythagorean theorem.

the pythagorean theorem formula a^2+b^2=c^2 where c is the hypotenuse and sides a and b are the shorter sides known as the legs of the right triangle

If we have a right triangle, and we construct squares using the edges or sides of the right triangle (gray triangle in the middle), the area of the largest square built on the hypotenuse (the longest side) is equal to the sum of the areas of the squares built on the other two sides. This is the Pythagorean Theorem in a nutshell. By the way, this is also known as the Pythagoras’ Theorem .

Pythagorean theorem illustrated using areas of squares

Notice that we square (raised to the second power) the variables [latex]a[/latex], [latex]b[/latex], and [latex]c[/latex] to indicate areas. The sum of the smaller squares (orange and yellow) is equal to the largest square (blue).

The Pythagorean Theorem relates the three sides in a right triangle. To be specific, relating the two legs and the hypotenuse, the longest side.

The Pythagorean Theorem can be summarized in a short and compact equation as shown below.

Pythagorean Theorem is simply summarized by the equation c squared equals a squared plus b squared

Definition of Pythagorean Theorem

For a given right triangle, it states that the square of the hypotenuse, [latex]c[/latex], is equal to the sum of the squares of the legs, [latex]a[/latex] and [latex]b[/latex]. That is, [latex]{a^2} + {b^2} = {c^2}[/latex].

For a more general definition, we have:

In right a triangle, the square of longest side known as the hypotenuse is equal to the sum of the squares of the other two sides.

The Pythagorean Theorem guarantees that if we know the lengths of two sides of a right triangle, we can always determine the length of the third side.

Here are the three variations of the Pythagorean Theorem formulas:

pythagorean theorem formula is c equals square root of a^2 plus b^2

Let’s go over some examples!

Examples of Applying the Pythagorean Theorem

Example 1: Find the length of the hypotenuse.

right triangle with legs of 7 inches and 5 inches

Our goal is to solve for the length of the hypotenuse. We are given the lengths of the two legs. We know two sides out of the three! This is enough information for the formula to work.

For the legs, it doesn’t matter which one we assign for [latex]a[/latex] or [latex]b[/latex]. The result will be the same. So if we let [latex]a=5[/latex], then [latex]b=7[/latex]. Substituting these values into the Pythagorean Formula equation, we get

c equals square root of 74

To isolate the variable [latex]c[/latex], we take the square roots of both sides of the equation. That eliminates the square (power of 2) on the right side. And on the left, we simply have a square root of a number which is no big deal.

However, we need to be mindful here when we take the square root of a number. We want to consider only the principal square root or the positive square root since we are dealing with length. It doesn’t make any sense to have a negative length, thus we disregard the negative length!

Therefore, the length of the hypotenuse is [latex]\sqrt {74}[/latex] inches. If we wish to approximate it to the nearest tenth, we have [latex]8.6[/latex] inches.

Example 2: Find the length of the leg.

right triangle with hypotenuse of 9 cm and leg of 7 cm

Just by looking at the figure above, we know that we have enough information to solve for the missing side. The reason is the measure of the two sides are given and the other leg is left as unknown. That’s two sides given out of the possible three.

Here, we can let [latex]a[/latex] or [latex]b[/latex] equal [latex]7[/latex]. It really doesn’t matter. So, for this, we let [latex]a=7[/latex]. That means we are solving for the leg [latex]b[/latex]. But for the hypotenuse, there’s no room for error. We have to be certain that we are assigning [latex]c[/latex] for the length, that is, for the longest side. In this case, the longest side has a measure of [latex]9[/latex] cm and that is the value we will assign for [latex]c[/latex], therefore [latex]c=9[/latex].

Let’s calculate the length of leg [latex]b[/latex]. We have [latex]a=7[/latex] and [latex]c=9[/latex].

c equals 4 times square root of 2

Therefore, the length of the missing leg is [latex]4\sqrt 2[/latex] cm. Rounding it to two decimal places, we have [latex]5.66[/latex] cm.

Example 3: Do the sides [latex]17[/latex], [latex]15[/latex] and [latex]8[/latex] form a right triangle? If so, which sides are the legs and the hypotenuse?

If these are the sides of a right triangle then it must satisfy the Pythagorean Theorem. The sum of the squares of the shorter sides must be equal to the square to the longest side. Obviously, the sides [latex]8[/latex] and [latex]15[/latex] are shorter than [latex]17[/latex] so we will assume that they are the legs and [latex]17[/latex] is the hypotenuse. So we let [latex]a=8[/latex], [latex]b=15[/latex], and [latex]c=17[/latex].

Let’s plug these values into the Pythagorean equation and check if the equation is true.

289 equals 289

Since we have a true statement, then we have a case of a right triangle! We can now say for sure that the shorter sides [latex]8[/latex] and [latex]15[/latex] are the legs of the right triangle while the longest side [latex]17[/latex] is the hypotenuse.

Example 4: A rectangle has a length of [latex]8[/latex] meters and a width of [latex]6[/latex] meters. What is the length of the diagonal of the rectangle?

The diagonal of a rectangle is just the line segment that connects two non-adjacent vertices. In the figure below, it is obvious that the diagonal is the hypotenuse of the right triangle while the two other sides are the legs which are [latex]8[/latex] and [latex]6[/latex].

rectangle with length of 8 meters and width of 8 meters

If we let [latex]a=6[/latex] and [latex]b=8[/latex], we can solve for [latex]c[/latex] in the Pythagorean equation which is just the diagonal.

c=10

Therefore, the measure of the diagonal is [latex]10[/latex] meters.

Example 5: A ladder is leaning against a wall. The distance from the top of the ladder to the ground is [latex]20[/latex] feet. If the base of the ladder is [latex]4[/latex] feet away from the wall, how long is the ladder?

ladder leaning against a wall

If you study the illustration, the length of the ladder is just the hypotenuse of the right triangle with legs [latex]20[/latex] feet and [latex]4[/latex] feet.

Again, we just need to perform direct substitution into the Pythagorean Theorem formula using the known values then solve for [latex]c[/latex] or the hypotenuse.

c equals 4 times square root of 26

Therefore, the length of the ladder is [latex]4\sqrt {26}[/latex] feet or approximately [latex]20.4[/latex] feet.

Example 6: In a right isosceles triangle, the hypotenuse measures [latex]12[/latex] feet. What is the length of each leg?

Remember that a right isosceles triangle is a triangle that contains a 90-degree angle and two of its sides are congruent.

In the figure below, the hypotenuse is [latex]12[/latex] feet. The two legs are both labeled as [latex]x[/latex] since they are congruent.

a right isosceles triangle with hypotenuse of 12 feet and legs of x

Let’s substitute these values into the formula then solve for the value of [latex]x[/latex]. We know that [latex]x[/latex] is just the leg of the right isosceles triangle which is the unknown that we are trying to solve for.

x equals 6 times square root of 2

Therefore, the leg of the right isosceles triangle is [latex]6\sqrt 2[/latex] feet. If we want an approximate value, it is [latex]8.49[/latex] feet, rounded to the nearest hundredth.

Example 7: The diagonal of the square below is [latex]2\sqrt 2[/latex]. Find its area.

square with a diagonal of 2 times square root of 2

We know the area of the square is given by the formula [latex]A=s^2[/latex] where [latex]s[/latex] is the side of the square. So that means we need to find the side of the square given its diagonal. If we look closely, the diagonal is simply the hypotenuse of a right triangle. More importantly, the legs of the right triangle are also congruent.

Since the legs are congruent, we can let it equal to [latex]x[/latex].

right triangle with hypotenuse of 2 times square root of 2 and legs of x

Substitute these values into the Pythagorean Theorem formula then solve for [latex]x[/latex].

x equals 2

We calculated the length of the leg to be [latex]2[/latex] units. It is also the side of the square. So to find the area of the square, we use the formula

[latex]A = {s^2}[/latex]

That means, the area is

[latex]A = {s^2} = {\left( 2 \right)^2} = 4[/latex]

Therefore, the area of the square is [latex]4[/latex] square units.

You might also like these tutorials:

  • Pythagorean Theorem Practice Problems with Answers
  • Pythagorean Triples
  • Generating Pythagorean Triples

Pythagorean Theorem and Problems with Solutions

Explore some simple proofs of the Pythagorean theorem and its converse and use them to solve problems. Detailed solutions to the problems are also presented.


In the figure below are shown two squares whose sides are a + b and c. let us write that the area of the large square is the area of the small square plus the total area of all 4 congruent right triangles in the corners of the large square.


(a + b) = c + 4 (1 / 2) (a b)
Expand the left hand side of the above equality, and simplify the last term on the right
a + b + 2 a b = c + 2 a b
Simplify to obtain
+ b = c a and b are the sides of the right triangle and c is its hypotenuse.



We first start with a right triangle and then complete it to make a rectangle as shown in the figure below which in turn in made up of three triangles.


The fact that the sides of the rectangle are parallel, that gives rise to angles being congruent (equal in size) in all three triangles shown in the figure which leads to the triangles being similar. We first consider triangles ABE and AED which are similar because of the equality of angles. The proportionality of corresponding sides (a in triangle ABE corresponds to c in triangle AED since both faces a right angle, x in triangle ABE corresponds to a in triangle AED both faces congruent (equal) angles) gives
a / c = x / a which may be written as a = c x
We next consider triangles ECD and AED which are similar because of the equality of angles. The corresponding sides are proportional, hence
b / c = y / b which may be written as b = c y
We now add the sides of the equalities a = c x and b = c y to obtain
a + b = c x + c y = c (x + y)
Use the fact that c = x + y to write
a + b = c The converse of the Pythagorean theorem sates that: if a, b and c are the lengths of a triangle with c the longest side and a + b = c then this triangle is a right triangle and c is the length of its hypotenuse.



Find the area of a right triangle whose hypotenuse is equal to 10 cm and one of its sides is 6 cm.


Which of the following may be the lengths of the sides of a right angled triangle?
a) (2 , 3 , 4)    b) (12 , 16 , 20)    c) (3√2 , 3√2 , 6)


The points A(0 , 2), B(-2 , -1) and C(- 1 , k) are the vertices of a right triangle with hypotenuse AB. Find k.


One side of a triangle has a length that is 2 meters less that its hypotenuse and its second side has a length that is 4 meters less that its hypotenuse. Find the perimeter of the triangle.


Find the perimeter of an equilateral triangle whose height is 60 cm.


Calculate the are of a square field whose diagonal is 100 meters.


Find x in the two right triangles figure below.




Given the hypotenuse and one of the sides, we use the Pythagorean theorem to find the second side x as follows


x + 6 = 10
Solve for x
x = √ (10 - 6 ) = 8
Area of the triangle = (1 / 2) height × base
The two sides of a right triangle make a right angle and may therefore be considered as the height and the base. Hence
Area of the triangle = (1 / 2) 6 × 8 = 24 cm


We use the converse of the Pythagorean theorem to solve this problem.
a) (2 , 3 , 4) : 4 is the length of the longest side
2 + 3 = 13
4 = 16
since 2 + 3 is NOT EQUAL to 4 , (2 , 3 , 4) are not the lengths of the sides of a right triangle.

b) (12 , 16 , 20) : 20 is the longest side
12 + 16 = 400
20 = 400
since 12 + 16 is EQUAL to 20 , (12 , 16 , 20) are the lengths of the sides of a right triangle.

c) (3√2 , 3√2 , 6) : 6 is the longest side
(3√2) + (3√2) = 36
6 = 36
(3√2) + (3√2) is EQUAL to 6 , therefore (3√2 , 3√2 , 6) are the length of a right triangle with the length of the hypotenuse equal to 6. Also since two sides have equal length to 3√2, the right triangle is isosceles.


The points A(0 , 2), B(-2 , -1) and C(- 1 , k) are the vertices of a right triangle with hypotenuse AB. Find k.
Use the formula to find the distance squared between two points given by d = (x - x ) + (y - y ) to find the square of the length of the hypotenuse AB and the sides AC and BC.
AB = (-2 - 0) + (-1 - 2) = 13
AC = (-1 - 0) + (k - 2) = k - 4 k + 5
BC = (-1 - (-2)) + (k - (-1)) = k +2 k + 2
AB is the hypotenuse of the triangle, we now use the Pythagorean theorem AB = AC + BC to obtain an equation in k.
13 = k - 4 k + 5 + k + 2 k + 2
2 k - 2 k - 6 = 0
k - k - 3 = 0
Solve for k the above quadratic equation to obtain two solutions k and k given by
k = ( 1 + √ (13) ) / 2 ? 2.30 and k = ( 1 - √ (13) ) / 2 ? - 1.30
There are two possible points C and C that make a right triangle with A and B and whose coordinates are given by:
C (- 1 , 2.30) and C (- 1 , - 1.30)
The graphical interpretation of the solution to this problem is shown below. Solution k corresponds to the "red" right triangle and solution k corresponds to the "blue" right triangle.




Let x be the hypotenuse of the right triangle. One side is x - 2 and the other is x - 4 as shown below. We need to find in order to find that sides and then the perimeter.

Use the Pythagorean to write
x = (x - 2) + (x - 4)
Expand and group to obtain the quadratic equation
x - 12 x + 20 = 0
Solve to find two solutions
x = 2 and x = 10
Calculate the sides and perimeter for each solution
hypotenuse: x = 2 , side 1 : x - 2 = 0 and side 2 : x - 4 = - 2
The sides of a triangle cannot be zero or negative and therefore x = 2 is not a solution to the given problem.
hypotenuse: x = 10 , side 1 : x - 2 = 8 and side 2 : x - 4 = 6
Perimeter = hypotenuse + side 1 + side 2 = 10 + 8 + 6 = 24 units.


An equilateral triangle with side x is shown below with height CH = 60 cm. Since it is an equilateral the height CH split the base (segment) AB into two equal segments of size x / 2.

We use the Pythagorean theorem on triangle CHB (or CHA) to write
x = 60 + ( x / 2)
Expand the above equation and rewrite as
3 x / 4 = 3600
Solve for x and take the positive solution since x is the size of the side of the triangle and must be positive.
x = 40 √3 cm
Perimeter = 3 x = 120 √3 cm


A square of side x and diagonal 100 m is shown below.
Area of the square = x
Use the Pythagorean theorem on the triangle ACD to write
x + x = 100
Solve for x
Area of square = x = 5000 m




We first use the Pythagorean theorem to the right triangle ECD in order to find the length of CD
5 = 3 + CD
Solve for CD
CD = √(25 - 9) = 4
We also know that
FC + CD = 6
Hence
FC = 6 - CD = 2
We now use the Pythagorean theorem to the right triangle EFC to write
x = 3 + FC = 9 + 4 = 13
Solve for
x = √(13)




pythagoras theorem problem solving

Advertisement

How The Pythagorean Theorem Helps Solve a Right Triangle

  • Share Content on Facebook
  • Share Content on LinkedIn
  • Share Content on Flipboard
  • Share Content on Reddit
  • Share Content via Email

pythagoras theorem problem solving

Key Takeaways

  • The Pythagorean theorem is crucial in various fields, including construction, manufacturing and navigation, enabling precise measurements and the creation of right angles for large structures.
  • It underpins our entire system of measurement, allowing for accurate navigation by pilots and ships, and making GPS measurements possible through the calculation of distances and angles.
  • Beyond navigation, the theorem is essential in geometry, physics, geology, engineering and even practical applications by carpenters and machinists.

The Pythagorean theorem is an ancient mathematical theorem which is one of the most fundamental and important concepts in two-dimensional Euclidean geometry going back thousands of years. It can help students find the sides of a right triangle on a piece of paper, but it also has greater implications in the fields of engineering, physics and architecture.

Since triangles always follow concrete rules we can use concepts like the Pythagorean theorem formula — and later, trigonometry — to find all the parameters of a triangle (angle values, lengths) if at least one of them is known.

The Pythagorean theorem is the simplest of these concepts and lets us easily solve the length of a third side of a right triangle if two sides are currently known.

Basics of The Pythagorean Theorem

Solving a right angled triangle using pythagorean formula, common forms of pythagorean triples, who was pythagoras, pythagorean theorem calculator, how is the pythagorean theorem useful today.

In its simplest form , the Pythagorean theorem states that in a hypothetical right triangle abc : a² + b² = c² .

The value of c² is equal to the sum of the squares, where hypotenuse c is the longest side of a right triangle. It's also always the side opposite the right angle.

Using this formula, we can always find the length of the hypotenuse if the other two sides are known values. After adding the numbers, we will need to apply a square root operation to arrive at the value of c .

Going back to triangle abc , what do we do if one of the known sides is the hypotenuse? We can reverse the Pythagorean theorem formula and turn it into a subtraction problem, then apply a square root just as before.

If a triangle contains two unknown sides, then more complex trigonometric formulas and algebraic proofs will have to be applied in order to find them. This same mathematical theorem can also be applied to physics problems like triangular force vectors.

What Is a Right Triangle?

A right angled triangle has exactly one of its angle values equal to 90 degrees, which is where the Pythagorean theorem formula can be applied. The side opposite the right angle is known as the hypotenuse and will always be the longest side of the right angled triangle.

Triangles without a right angle, like a scalene or isosceles triangle, cannot be solved using the Pythagorean theorem. They must be broken up into smaller shapes or have more complex formulas applied.

Like all triangles, the angle values of a right angled triangle add up to a sum of 180 degrees. This also means that the two non-right angles of the triangle must add up to 90 degrees.

Now that we know a bit about solving right angled triangle abc , let's replace our variables with real numbers and run through the formula again. The side lengths we know are 16 and 20, and our hypotenuse is the unknown side.

Based on these calculations, we now know that the hypotenuse of the triangle equals 25.61.

Another useful idea related to Pythagoras theorem proof is the concept of a Pythagorean triple. These are essentially forms of right triangles which have sides that are all equal to whole numbers.

The most common form of a Pythagorean triple you are likely to see math education is known as the (3, 4, 5) triangle. If two sides of a right triangle equal 3 and 4, then the hypotenuse will always be 5.

Shown using the Pythagorean theorem formula:

Learning to spot Pythagorean triples by eye can help you easily solve them without resorting to the Pythagorean theorem formula every time. There are theoretically infinite Pythagorean triples out there, but some other common ones include:

Similar triangles to a Pythagorean triple will themselves be triples. So we can multiply all the values of our previous example by 2 to get a triangle of (6, 8, 10).

Multiplying any Pythagorean triple by any positive integer (as in, applying the same multiplier to all sides of the same triangle) will give you similar results.

The Pythagorean equation is most often attributed to Pythagoras of Somos , but we now know that many ancient civilizations like those in Egypt, India and China had discovered the mathematical relationship independently.

That said, the man whom this math trick is named for is nearly as fascinating. Pythagoras, an ancient Greek thinker who was born on the island of Samos and lived from 570 to 490 B.C.E, was kind of a trippy character — equal parts philosopher, mathematician and mystical cult leader.

In his lifetime, Pythagoras wasn't known as much for solving for the length of the hypotenuse as he was for his belief in reincarnation and adherence to an ascetic lifestyle that emphasized a strict vegetarian diet, adherence to religious rituals and plenty of self-discipline that he taught to his followers.

Pythagoras biographer Christoph Riedweg describes him as a tall, handsome and charismatic figure, whose aura was enhanced by his eccentric attire — a white robe, trousers and a golden wreath on his head. Odd rumors swirled around him — that he could perform miracles, that he had a golden artificial leg concealed beneath his clothes and that he possessed the power to be in two places at one time.

Pythagoras founded a school near what is now the port city of Crotone in southern Italy, which was named the Semicircle of Pythagoras. Followers, who were sworn to a code of secrecy, learned to contemplate numbers in a fashion similar to the Jewish mysticism of Kaballah. In Pythagoras' philosophy, each number had a divine meaning, and their combination revealed a greater truth.

With a hyperbolic reputation like that, it's little wonder that Pythagoras was credited with devising one of the most famous theorems of all time, even though he wasn't actually the first to come up with the concept. Chinese and Babylonian mathematicians beat him to it by a millennium.

"What we have is evidence they knew the Pythagorean relationship through specific examples," writes G. Donald Allen , a math professor and director of the Center for Technology-Mediated Instruction in Mathematics at Texas A&M University, in an email. "An entire Babylonian tablet was found that shows various triples of numbers that meet the condition: a 2 + b 2 = c 2 ."

The earliest known example of the Pythagorean theorem formula is on a clay tablet unearthed in modern-day Iraq and now resides in a museum in Istanbul. This tablet of Babylonian origin displays various trigonometric functions, including what we now know as the Pythagorean theorem, but it predates Pythagoras by more than 1,000 years. Historians estimate the tablet was drawn as early as 1,900 B.C.E.

The Pythagorean theorem isn't just an intriguing mathematical exercise. It's utilized in a wide range of fields, from construction and manufacturing to navigation.

As Allen explains, one of the classic uses of the Pythagorean theorem is in laying the foundations of buildings. "You see, to make a rectangular foundation for, say, a temple, you need to make right angles. But how can you do that? By eyeballing it? This wouldn't work for a large structure. But, when you have the length and width, you can use the Pythagorean theorem to make a precise right angle to any precision."

Beyond that, "This theorem and those related to it have given us our entire system of measurement," Allen says. "It allows pilots to navigate in windy skies, and ships to set their course. All GPS measurements are possible because of this theorem."

In navigation, the Pythagorean theorem provides a ship's navigator with a way of calculating the distance to a point in the ocean that's, say, 300 miles north and 400 miles west (480 kilometers north and 640 kilometers west). It's also useful to cartographers, who use it to calculate the steepness of hills and mountains.

"This theorem is important in all of geometry, including solid geometry," Allen continues. "It is also foundational in other branches of mathematics, much of physics, geology, all of mechanical and aeronautical engineering. Carpenters use it and so do machinists. When you have angles, and you need measurements, you need this theorem."

One of the formative experiences in the life of Albert Einstein was writing his own mathematical proof of the Pythagorean theorem at age 12. Einstein's fascination with geometry eventually played a role in his development of the theories of special and general relativity.

Please copy/paste the following text to properly cite this HowStuffWorks.com article:

number 33, mathematics problem

  • International
  • Education Jobs
  • Schools directory
  • Resources Education Jobs Schools directory News Search

Problem solving using Pythagoras' Theorem

Problem solving using Pythagoras' Theorem

Subject: Mathematics

Age range: 14-16

Resource type: Worksheet/Activity

Phil Rhodes

Last updated

8 March 2020

  • Share through email
  • Share through twitter
  • Share through linkedin
  • Share through facebook
  • Share through pinterest

docx, 6.06 MB

5 Problems that require careful thinking using Pythagoras’ Theorem. Suitable for anyone who has already covered the basics Answers included on P2

Creative Commons "Sharealike"

Your rating is required to reflect your happiness.

It's good to leave some feedback.

Something went wrong, please try again later.

anniecusack

Empty reply does not make any sense for the end user

Report this resource to let us know if it violates our terms and conditions. Our customer service team will review your report and will be in touch.

Not quite what you were looking for? Search by keyword to find the right resource:

arXiv's Accessibility Forum starts next month!

Help | Advanced Search

Mathematics > Optimization and Control

Title: numerical exploration of the pythagorean theorem using hobo algorithm.

Abstract: This paper introduces a novel method for finding integer sets that satisfy the Pythagorean theorem by leveraging the Higher-Order Binary Optimization (HOBO) formulation. Unlike the Quadratic Unconstrained Binary Optimization (QUBO) formulation, which struggles to express complex mathematical equations, HOBO's ability to model higher-order interactions between binary variables makes it well-suited for addressing more complex and expressive problem settings.
Subjects: Optimization and Control (math.OC)
Cite as: [math.OC]
  (or [math.OC] for this version)
  Focus to learn more arXiv-issued DOI via DataCite

Submission history

Access paper:.

  • HTML (experimental)
  • Other Formats

license icon

References & Citations

  • Google Scholar
  • Semantic Scholar

BibTeX formatted citation

BibSonomy logo

Bibliographic and Citation Tools

Code, data and media associated with this article, recommenders and search tools.

  • Institution

arXivLabs: experimental projects with community collaborators

arXivLabs is a framework that allows collaborators to develop and share new arXiv features directly on our website.

Both individuals and organizations that work with arXivLabs have embraced and accepted our values of openness, community, excellence, and user data privacy. arXiv is committed to these values and only works with partners that adhere to them.

Have an idea for a project that will add value for arXiv's community? Learn more about arXivLabs .

IMAGES

  1. 😀 Pythagorean theorem problem solving. Tenth grade Lesson Using the

    pythagoras theorem problem solving

  2. Pythagoras Theorem

    pythagoras theorem problem solving

  3. 72 Problem Solving using Pythagoras Theorem

    pythagoras theorem problem solving

  4. Pythagorean Theorem Word Problems

    pythagoras theorem problem solving

  5. Problem solving using Pythagoras' Theorem

    pythagoras theorem problem solving

  6. Pythagoras: solving problems using Pythagoras' theorem

    pythagoras theorem problem solving

COMMENTS

  1. Pythagorean Theorem Practice Problems With Answers

    Here are eight (8) Pythagorean Theorem problems for you to solve. You might need to find either the leg or the hypotenuse of the right triangle. These problems vary in type and difficulty, providing you an opportunity to level up your skills.

  2. 15 Pythagorean Theorem Practice Problems For 8th Grade

    Pythagorean Theorem practice problems involve using the relationship between the sides of a right triangle to calculate missing side lengths in triangles. The Pythagorean Theorem is introduced in 8th grade and is used to solve a variety of problems across high school.

  3. Pythagorean Theorem: Problems with Solutions

    Calculate the total area of the plot. Answer: square kilometers. It is given the length of the diagonal of the square. It divides it into two equal triangles. In addition, the two triangles are right and the legs of the same length. be the length of square side and by the Pythagorean theorem we get: \displaystyle x^ {2} +x^ {2}=\left (2\sqrt {2 ...

  4. Pythagoras Theorem Questions

    Here you will find our Pythagoras Theorem Questions page. Find out all about Pythagoras' theorem and how it can be used in different contexts by the Math Salamanders.

  5. Word problems on Pythagorean Theorem

    Learn how to solve different types of word problems on Pythagorean Theorem. Pythagoras Theorem can be used to solve the problems step-by-step when we know the length of two sides

  6. Pythagorean Theorem

    The Pythagorean Theorem is one of the most frequently used theorems in geometry, and is one of the many tools in a good geometer's arsenal. A very large number of geometry problems can be solved by building right triangles and applying the Pythagorean Theorem. This is generalized by the Pythagorean Inequality and the Law of Cosines .

  7. Pythagoras Theorem Questions [Solved]

    Pythagoras theorem practice questions with detailed solutions are provided for practice purposes. Visit BYJU'S today to solve Pythagoras theorem questions and explore study resources.

  8. Pythagorean Theorem

    Free pythagorean theorem math topic guide, including step-by-step examples, free practice questions, teaching tips and more!

  9. Pythagoras Practice Questions

    The Corbettmaths Practice Questions on Pythagoras

  10. The Pythagorean theorem with examples

    The Pythagorean theorem and how to use it to find the hypotenuse, sides of a right triangle, and unknown values in word problems (applications).

  11. Geometry: Pythagorean Theorem

    Pythagorean Theorem - How to use the Pythagorean Theorem, Converse of the Pythagorean Theorem, Worksheets, Proofs of the Pythagorean Theorem using Similar Triangles, Algebra, Rearrangement, How to use the Pythagorean Theorem to solve real-world problems, in video lessons with examples and step-by-step solutions.

  12. Pythagorean Theorem

    Learn about the Pythagorean theorem in geometry. Get its definition, formula, and worked example math problems.

  13. Pythagorean Theorem

    How to use the pythagorean theorem, explained with examples, practice problems, a video tutorial and pictures.

  14. Using the Pythagorean Theorem to Solve Problems

    Learning Outcomes Use the pythagorean theorem to find the unknown length of a right triangle given the two other lengths The Pythagorean Theorem is a special property of right triangles that has been used since ancient times. It is named after the Greek philosopher and mathematician Pythagoras who lived around [latex]500 [/latex] BCE.

  15. Pythagorean Theorem Word Problems

    How to use the Pythagorean Theorem to solve Word Problems, how to solve different types of word problems using the Pythagorean Theorem, real life Pythagorean Theorem word problems, grade 9, grade 8, in video lessons with examples and step-by-step solutions.

  16. How to Solve Pythagorean Theorem Problems? (+FREE Worksheet!)

    How to Solve Pythagorean Theorem Problems? (+FREE Worksheet!) In mathematics, the Pythagorean Theorem is the relationship between three sides of a right triangle.

  17. Pythagorean theorem challenge (practice)

    Do 4 problems. Learn for free about math, art, computer programming, economics, physics, chemistry, biology, medicine, finance, history, and more. Khan Academy is a nonprofit with the mission of providing a free, world-class education for anyone, anywhere.

  18. Pythagorean Theorem

    What is the Pythagorean theorem & what is it used for - learn how to solve it with equation, proofs, real-life examples, solved problems, and diagram

  19. Pythagorean Theorem

    The Pythagorean Theorem can be summarized in a short and compact equation as shown below. For a given right triangle, it states that the square of the hypotenuse, In right a triangle, the square of longest side known as the hypotenuse is equal to the sum of the squares of the other two sides. The Pythagorean Theorem guarantees that if we know ...

  20. Pythagorean theorem word problems (practice)

    Learn for free about math, art, computer programming, economics, physics, chemistry, biology, medicine, finance, history, and more. Khan Academy is a nonprofit with the mission of providing a free, world-class education for anyone, anywhere.

  21. Pythagorean Theorem and Problems with Solutions

    Simple proofs of the Pythagorean theorem and right angled triangle problems with detailed solutions are presented.

  22. How The Pythagorean Theorem Helps Solve a Right Triangle

    The Pythagorean equation is most often attributed to Pythagoras of Somos, but we now know that many ancient civilizations like those in Egypt, India and China had discovered the mathematical relationship independently.. That said, the man whom this math trick is named for is nearly as fascinating. Pythagoras, an ancient Greek thinker who was born on the island of Samos and lived from 570 to ...

  23. Problem solving using Pythagoras' Theorem

    Problem solving using Pythagoras' Theorem. Subject: Mathematics. Age range: 14-16. Resource type: Worksheet/Activity. File previews. docx, 6.06 MB. 5 Problems that require careful thinking using Pythagoras' Theorem. Suitable for anyone who has already covered the basics. Answers included on P2.

  24. [2408.11076v1] Numerical Exploration of the Pythagorean Theorem Using

    This paper introduces a novel method for finding integer sets that satisfy the Pythagorean theorem by leveraging the Higher-Order Binary Optimization (HOBO) formulation. Unlike the Quadratic Unconstrained Binary Optimization (QUBO) formulation, which struggles to express complex mathematical equations, HOBO's ability to model higher-order interactions between binary variables makes it well ...